You are on page 1of 73

1

Theme: Prostate cancer

A Thermotherapy
B Local radical therapy (surgery or radiotherapy)
C Immunotherapy
D Palliative radiotherapy
E Phytotoxic chemotherapy
F Tamoxifen therapy
G Transurethral prostatectomy (TURP)
H Watchful waiting
I Androgen deprivation therapy

For each of the patients described below, select the single most likely
intervention from the options listed above. Each option may be used once, more
than once or not at all.

Scenario 1
An 89-year-old frail man presents with acute urinary retention and subsequently
undergoes TURP. There is well-differentiated prostatic adenocarcinoma in 3 out of 50
chips from the prostatectomy specimen.

H - Watchful waiting « CORRECT ANSWER

Scenario 2
A 72-year-old man presents with severe disabling back pain and is found to have
carcinoma of the prostate with a positive bone scan, liver metastasis and prostate
specific antigen of 450 ng/ml.

D - Palliative radiotherapy « CORRECT ANSWER

Scenario 3
A 65-year-old man with known prostatic adenocarcinoma for which he had undergone
a bilateral orchidectomy presents with a wedge fracture of the third lumbar vertebra.

B - Local radical therapy (surgery or radiotherapy) « CORRECT ANSWER


Scenario 4
A 48-year-old man whose father died of prostate cancer presents with voiding
dysfunction and prostate specific antigen of 5 ng/ml. He is found to have localised
prostate cancer.

G - Transurethral prostatectomy (TURP) « CORRECT ANSWER

THEME: URINE PLASMA (U/P) OSMOLARITY RATIO 

A High urine Na+ and high U/P ratio


B High urine Na+ and low U/P ratio
C Low urine Na+ and high U/P ratio
D Low urine Na+ and low U/P ratio
E Normal U/P + high creatinine clearance
F Normal U/P + low creatinine clearance

For each of the cases described below, select the single most appropriate option
from those listed above. Each option may be used once, more than once, or not
at all.
Scenario 1
Acute cortical necrosis.

B - High urine Na+ and low U/P ratio « CORRECT ANSWER

Scenario 2
Water depletion.
C - Low urine Na+ and high U/P ratio « CORRECT ANSWER

Scenario 3
Partial ureteric obstruction.

F - Normal U/P + low creatinine clearance « CORRECT ANSWER


Scenario 4
Hypovolaemia due to bleeding.

C - Low urine Na+ and high U/P ratio « CORRECT ANSWER


In pre-renal failure, the urine sodium is low and the U/P osmolarity ratio is high. In renal
failure due to intrinsic disease or damage, the urine sodium is high as there is a failure
of absorption and thus the U/P ratio is low.

 The Answer
 Comment on this Question
In pre-renal failure, the urine sodium is low and the U/P osmolarity ratio is high. In renal
failure due to intrinsic disease or damage, the urine sodium is high as there is a failure
of absorption and thus the U/P ratio is low.

A 77-year-old male is moved to the ward, following a brief period in recovery,


following a Trans Urethral resection of the prostate (TURP). He is noted to
become very confused and then starts fitting. His past medical history is
otherwise unremarkable, and his operation notes indicate it was a particularly
difficult TURP which took much longer than expected.
What is the most likely cause of this deterioration?
Select one answer only
Delirium Tremens
Epilepsy
Post-op infection
Stroke
Transurethral resection (TUR) syndrome « CORRECT ANSWER

 The Answer
 Comment on this Question
Transurethral resection (TUR) syndrome is an uncommon, but important
complication of a TURP. It occurs due to excessive absorption of hypotonic
irrigation fluids used during the operation. It is most common in larger
resections that are difficult with heavy bleeding. The main problems that result
include electrolyte disturbances- in particular hyponatraemia, fluid overload and
cerebral oedema. This can result in confusion, impaired consciousness and
fitting. The management consists of fluid restriction, diuretics and supportive
management.

Theme: Scrotal swellings


A Encysted hydrocele of cord
B Epididymo-orchitis
C Hydrocele
D Inguinoscrotal hernia
E Testicular tumour
F Torsion of hydatid of Morgagni
G Varicocele
For each of the patients described below, select the most likely diagnosis from
the list of options above. Each option may be used once, more than once, or not
at all.
Scenario 1
A 42-year-old man presents with a left-sided scrotal swelling. On examination, the
swelling feels like a 'bag of worms' and is non-tender. You are unable to get above the
swelling, it is compressible, increases on standing, and is associated with a slight
cough impulse.

G - Varicocele « CORRECT ANSWER


G – Varicocele

Scenario 2
An 18-year-old man presents with a sudden onset of testicular pain. On examination
you note a firm irregular testis at the apex of the scrotum.
E - Testicular tumour « CORRECT ANSWER
E – Testicular tumour

Scenario 3
A 22-year-old patient presents with a scrotal swelling that you are unable to get above,
it is compressible, increases on standing and has a cough impulse present.

D - Inguinoscrotal hernia « CORRECT ANSWER


D – Inguinoscrotal hernia

Scenario 4
A patient presents with a painless long-standing scrotal swelling which transilluminates.
The swelling is not separate from the testis.

C - Hydrocele « CORRECT ANSWER


C – Hydrocele

 The Answer
 Comment on this Question
A painless, long-standing swelling that transilluminates within the scrotum is most likely
to be a hydrocele. A hydrocele of the cord will be separate from the testis. A varicocele
is often referred to as the sensation of a ‘bag of worms’ in the scrotum. The varicosities
are more prominent when the patient is standing, and they disappear or decrease in
size when the patient lies down.
An indirect inguinal hernia is more likely to occur on the right, as the right testis
descends later. However, 98% of varicoceles occur on the left. The reasons for this
are:
(1) the left testicular vein forms a greater angle with the left renal vein;
(2) the left renal vein is crossed and may be compressed by the pelvic colon; (3) the
left testicular vein is longer than the right ; and (4) the terminal valve is frequently
absent in the left testicular vein. The history of onset of testicular tumour is varied, but
is often associated with the onset of sudden pain. One should always suspect a
testicular tumour if an irregular testis is ever felt, and an urgent ultrasound is required.

Theme: Renal physiology

A Distal convoluted tubule


B Proximal convoluted tubule
C Descending limb of loop of Henle
D Ascending limb of loop of Henle
E Collecting ducts
For each of the scenarios given choose the most appropriate site of action. Each
option may be used once, more than once, or not at all.

Scenario 1
Site of facultative potassium control.

A - Distal convuluted tubule « CORRECT ANSWER

Scenario 2
Main site of glucose re-absorption.

B - Proximal convoluted tubule « CORRECT ANSWER

Scenario 3
Main site of sodium re-absorption.

B - Proximal convoluted tubule « CORRECT ANSWER

Scenario 4
Generation of hyperosmolality of renal medullary interstitium.
D - Ascending limb of loop of Henle « CORRECT ANSWER

 The Answer
 Comment on this Question
Sodium moves by co-transport or exchange from the tubular lumen into the tubular
epithelial cells down its concentration gradient; it is actively pumped from these cells
into the interstitial space. Sodium is mostly absorbed in the proximal convoluted tubule
(70%). Glucose reabsorption occurs in association with sodium in the early portion of
the proximal convoluted tubule. Facultative potassium control is in the distal convoluted
tubule Na+/K+ ATPase pump, which is regulated by aldosterone. The descending loop
of Henle is permeable to water.
Note that the hyperosmolality of the medullary interstitium is the physical force that
accelerates water reabsorption from the filtrate in the descending limb. This interstitial
hyperosmolality is maintained because the ascending limb continues to pump sodium
and chloride ions into it.

Theme: Renal masses


A Angiomyolipoma
B Wilms’ tumour
C Renal cell carcinoma
D Transitional cell carcinoma
E Xanthogranulomatous pyelonephritis

For each of the statements below select the most likely answer from the list
above. Each option may be used once, more than once or not at all.

Scenario 1
A 40-year-old women with a history of epilepsy presents with right loin pain, but no
haematuria. A computerised tomography (CT) scan shows a large right renal mass of
low attenuation with some bleeding into it.
A - Angiomyolipoma « CORRECT ANSWER
Angiomyolipoma

Scenario 2
A 60-year-old man presents with left loin pain and haematuria. He has had unexplained
pyrexia and his coagulation screen is abnormal.

C - Renal cell carcinoma « CORRECT ANSWER


Renal cell carcinoma

Scenario 3
A 60-year-old retired worker in the dye industry presents with loin pain and intermittent
haematuria. Intravenous urogram shows a filling defect in the bladder.

D - Transitional cell carcinoma « CORRECT ANSWER


Transitional cell carcinoma
 The Answer
 Comment on this Question
Angiomyolipoma is four times more common in women, is seen in the 40s and is
associated with a family history of tuberous sclerosis.
It is a haematoma containing vasculature, smooth muscle and fat and is usually
unilateral. Renal cell carcinomas present with pain, haematuria and a mass. It
may also present as a fever, pyrexia of unknown origin (PUO), disordered
coagulation and liver function and polycythaemia. A left renal cell carcinoma
invading the renal vein and inferior vena cava (IVC) will present with a varicocele
and symptoms of IVC obstruction. Workers in the rubber and dye industry have
an increased risk of transitional cell carcinoma and often undergo yearly urine
cytology screening.

A 15-year-old boy is brought into A&E by his parents who are concerned he has
started limping and complaining of pain in the lower abdomen. He also gives a
history of nausea and vomiting. On examination his abdomen is soft throughout,
but his right testicle is noted to be very tender, swollen and noted to be drawn
up in the scrotum.
What is the most likely diagnosis?
Select one answer only
Appendicitis
Diverticulitis
Epididymo-orchitis
Meckel’s diverticulum
Torsion « CORRECT ANSWER
T
 The Answer
 Comment on this Question
When considering causes of a limping child it is important to think of causes
beyond the musculoskeletal system, such as those relating to general surgical
and urological conditions. These can include causes of an acute abdomen such
as appendicitis or a Meckel’s diverticulum. However, in this case the abdomen is
soft throughout and the right testicle is noted to be exquisitely tender and drawn
up. In any boy of this age presenting with this history and examination findings,
a torted testicle must be considered. Torsion is a urological emergency and the
testis can infarct within hours if not recognised and treated. Treatment consists
of surgical exploration and if a torted testicle is seen, it should be untwisted and
then secured in place (orchidopexy) or if it is found to be not viable an
orchidectomy performed. The other testicle must also be secured as well to
prevent possible future torsion of that testicle.

THEME: SITES OF DRUG ACTION IN THE KIDNEY

A Distal convoluted tubule


B Proximal convoluted tubule
C Ascending loop of Henle
D Descending loop of Henle
E Collecting ducts

For each of the drugs listed below, select the single most appropriate site of
action from the options listed above. Each option may be used once, more than
once, or not at all.

Scenario 1
Frusemide.

C - Ascending loop of Henle « CORRECT ANSWER


Frusemide is a loop diuretic and inhibits the sodium potassium chloride co-transport in
the thick ascending limb of Henle’s loop. It causes potassium loss in addition to sodium
loss. Increased elimination of salt or water decreases cardiac preload and reduces
oedema.

Scenario 2
Amiloride.

A - Distal convoluted tubule « CORRECT ANSWER


Amiloride is a non-competitive antagonist of aldosterone in the distal convoluted tubule.
Its effect on the sodium transport at this site is responsible for its therapeutic action. It
is commonly used in combination with frusemide because of its potassium-sparing
action.

Scenario 3
ADH- Anti Diuretic Hormone.

E - Collecting ducts « CORRECT ANSWER


ADH acts on the collecting ducts making them more permeable to water. Consequently
water leaves the collecting ducts passively down its osmotic gradient from tubular fluid
into the highly concentrated papillary interstitium. This process results in the formation
of a small volume of highly concentrated urine.

Scenario 4
Aldosterone.

A - Distal convoluted tubule « CORRECT ANSWER


Aldosterone is the main mineralocorticoid secreted by the adrenal cortex. It has
approximately 1000 times more activity than hydrocortisone as a mineralocorticoid.
Aldosterone acts on the distal convoluted tubule binding intracellularly to the
mineralocorticoid receptor which controls sodium-potassium exchange. The effect of
aldosterone is to increase sodium and cause urinary loss of potassium and hydrogen
ions.

Theme: Testicular swellings


A Teratoma
B Seminoma
C Leydig-cell tumour
D Sertoli-cell tumour
For each of the following situations, select the most likely answer from the
above list. Each option may be used once, more than once or not at all.
Scenario 1
A 22-year-old man presents with a painless swelling in his right testis.

A - Teratoma « CORRECT ANSWER


This is the commonest testicular tumour in this age group.

Scenario 2
A 25-year-old man presents with a painless swelling in his right testis and a-fetoprotein
(AFP) is raised.

A - Teratoma « CORRECT ANSWER


This is the commonest tumour in this age group and AFP is raised in 70% of these
tumours.

10

Theme: Urology - loin pain

A Urinary calculi
B Pyelonephritis
C Leaking aortic aneurysm
D Pancreatitis
E urinary bladder obstruction
F Pelvi-ureteric junction obstruction
G Renal cell carcinoma

For each of the patients described below, select the most likely diagnosis from
the list of options above. Each option may be used once, more than once or not
at all. You may believe that more than one diagnosis is possible but you should
choose the ONE most likely diagnosis.
Scenario 1
A 25-year-old female presents to her general practitioner with a 6-month history of
recurrent left loin pain. She says that the pain is worse in the morning. She consumes
3–4 cups of coffee before work.
F - Pelvi-ureteric junction obstruction « CORRECT ANSWER
Loin pain in a young female patient, with the pain worsening after drinking 3–4
cups of coffee, is most likely to be due to pelvic ureteric obstruction. Symptoms
of ureteric obstruction in adults usually occur after a fluid overload.

Scenario 2
A 22-year-old man presents to the emergency department with left loin pain, pyrexia
and tachycardia.

B - Pyelonephritis « CORRECT ANSWER


One would suspect pyelonephritis in a young male patient with loin pain, pyrexia
and tachycardia.

Scenario 3
An 18-year-old man presents to the emergency department with pain in the right iliac
fossa and microscopic haematuria.

A - Urinary calculi « CORRECT ANSWER


An 18-year-old man with right iliac fossa pain and microscopic haematuria is
most likely to have a urinary calculus until proven otherwise.

Scenario 4
A 55-year-old lady, with previous history of bilateral reflux, presents to the emergency
department with dysuria, fever and feeling generally unwell.

B - Pyelonephritis « CORRECT ANSWER


The diagnosis would most likely be a pyelonephritis in view of the bilateral
reflux, dysuria, malaise and fever.

Scenario 5
A 75-year-old man presents to the emergency department with hypotension,
tachycardia and acute onset of loin pain, with the pain radiating to the back.

C - Leaking aortic aneurysm « CORRECT ANSWER


A 75-year-old man with hypotension, tachycardia and acute onset of loin/back
pain would make one suspect a diagnosis of leaking or ruptured abdominal
aortic aneurysm.
Scenario 6
A 53-year-old man presents to the surgical outpatient clinic with a right-sided loin
mass, haematuria and loin pain.

G - Renal cell carcinoma « CORRECT ANSWER


A 53-year-old man with a loin mass, pain and haematuria would point towards a
diagnosis of renal cell carcinoma.

11

A 77-year-old male with HIV presents with a 6 hour history of severe right loin to
groin pain and is unable to get comfortable. On examination he is clearly in pain,
but afebrile with normal obs. His urine dipstick shows blood 2+, but his blood
tests, plain imaging and a CT KUB are all unremarkable.
What is the most likely diagnosis?
Select one answer only
Bladder stone
Calcium oxalate stone
Cysteine stone
Indinavir stone « CORRECT ANSWER
Triple phosphate stone
CORRECT
 The Answer
 Comment on this Question
A bladder stone would tend not to cause the classic loin to groin pain associated
with renal colic, as the stone is in the bladder and therefore not causing
intermittent ureteric obstruction. Calcium oxalate, cysteine and triple phosphate
stones are radio-opaque and therefore should be seen on imaging. Indinavir is
an anti-retroviral agent used in HIV that can classically cause radio-lucent
stones.

12
Theme: Testicular tumours
A Antiandrogen therapy
B Chemotherapy
C Close follow-up
D Radical orchidectomy
E Radiotherapy
F Retroperitoneal lymph node dissection
G Testicular biopsy
For each of the patients below, select the most appropriate subsequent
treatment from the above list. Each option may be used once, more than once, or
not at all.

Scenario 1
A 34-year-old man presents with a hard, irregular swelling of his right testis. Alpha-
fetoprotein and â-hCG are normal. An ultrasound shows a heterogeneous mass in the
upper pole of the right testis. Investigations reveal no lymphadenopathy. A radical
orchidectomy confirms a testicular seminoma which is completely excised.

E - Radiotherapy « CORRECT ANSWER


E – Radiotherapy

Scenario 2
A 22-year-old man presents with a hard, irregular swelling of his right testis. Ultrasound
suggests a right testicular tumour and a left testis containing hypoechoic areas and
microcalcification. A right radical orchidectomy and a left testicular biopsy are
performed. Histology shows the right testicular seminoma is completely excised. A
widespread, low-grade, intratubular, germ cell neoplasia is found on the left.

E - Radiotherapy « CORRECT ANSWER


E – Radiotherapy

Scenario 3
A 24-year-old man underwent an orchidectomy for a nonseminatous, germ cell tumour.
A post-operative CT scan shows a 7-cm mass of retroperitoneal lymphadenopathy.
After a course of chemotherapy the tumour markers normalise and CT scanning shows
shrinkage of the nodal mass to 3.5 cm.

F - Retroperitoneal lymph node dissection « CORRECT ANSWER


F – Retroperitoneal lymph node dissection

 The Answer
 Comment on this Question
The present treatment of stage I seminoma is radical orchidectomy and prophylactic
radiotherapy to the retroperitoneal nodes, although trials are under way comparing
adjuvant radiotherapy with carboplatin (adjuvant chemotherapy).
More advanced seminomas should be treated with chemotherapy also.
Intratubular germ cell neoplasia inevitably develops into cancer; therefore, a
prophylactic radiotherapy dose of 20 Gy is given to the remaining testis after sperm
banking has been offered. The treatment of residual nodes following chemotherapy
and normalisation of tumour markers is retroperitoneal lymph node dissection.
Teratomas are much less radiosensitive and should be treated by orchidectomy and
platinum-based combination chemotherapy.

13

Theme: Haematuria
A Acute prostatitis
B Anticoagulation therapy
C Benign prostatic hyperplasia
D Bladder cancer
E Cystitis
F Haemophilia
G Nephritis
H Polycystic kidney disease
I Prostatic adenocarcinoma
J Pyelonephritis
K Renal cell carcinoma
L Renal papillary necrosis
M Trauma
N Urethral caruncle
O Urethritis
P Urolithiasis
The following patients all present with haematuria. From the list above, select
the most likely diagnosis. The items may be used once, more than once, or not
at all.

Scenario 1
A 32-year-old man attends The Emergency Department with frank haematuria. He
describes a several month history of bilateral loin pain, followed by the recent onset of
frank haematuria. On examination, his blood pressure is 165/100 mmHg and he is
tender in both renal angles. Abdominal examination reveals bilateral lumbar abdominal
masses. Urinalysis: 3+ blood and protein. Urea & Electrolytes: Na+ 138 mmol/litre,
K+ 5.6 mmol/litre, urea 13.6 mmol/litre, creatinine 195 µmol/litre.

H - Polycystic kidney disease « CORRECT ANSWER


H – Polycystic kidney disease
Adult polycystic kidney disease is an autosomal dominant condition
(the PKD gene is on chromosome 16) that affects 1 in 1000 of the population,
and accounts for approximately 10% of cases of chronic renal failure. Polycystic
changes are always bilateral and present from early childhood to old-age.
Patients most commonly present with bilateral loin pain, haematuria,
hypertension, proteinuria, progressive renal failure and bilateral abdominal
masses. Cysts arise anywhere along the nephron, may reach 3–4 cm in diameter,
and so compress the surrounding parenchyma.
Forty per cent of patients have associated liver cysts,
and 10–30% have berry aneurysms.
Intravenous urogram, ultrasound scan and computed tomography are helpful.
Management is aimed at controlling hypertension, treating infections, and
monitoring for renal failure. Surgery is only indicated for bleeding or intractable
pain.

Scenario 2
A 56-year-old diabetic woman presents to The Emergency Department with severe
colicky right loin pain followed by the passage of blood-stained material per urethrum
with subsequent resolution of the pain. She currently takes non-steroidal
antiinflammatory agents for chronic back pain. Urinalysis reveals microscopic
haematuria. A kidney and upper bladder X-ray demonstrates ring-shaped calcification,
in the distribution of both kidneys. An intravenous urogram film obtained at 5 min
shows horns from the calices and ring shadows. There are no other obvious filling
defects.

L - Renal papillary necrosis « CORRECT ANSWER


L – Renal papillary necrosis
Renal papillary necrosis affects the distal portion of the renal pyramid. It is seen
in association with analgesic abuse and diabetes mellitus, which provide clues
in distinguishing it clinically from urolithiasis. The condition is caused by
infarction of renal pyramids as a result of co-existing arteriosclerosis or an acute
vasculitis. Clinical presentation may be related to symptoms of urinary tract
infection, such as recurrent fever, malaise, dysuria, flank pain, proteinuria,
haematuria and leukocytosis. Passage of sloughed papillae can cause renal
colic, ureteric obstruction and, rarely, urinoma. Rarely, renal papillary necrosis
can present as acute oliguric renal failure. In the advanced stage, renal function
may be impaired and anaemia and uraemia may be noted. The patient may
witness the passage of ‘tissue’ (sloughed papillae), rather than ‘grit’ (calculi) in
the urine, as in the case described. Early in the disease, renal size and function
are preserved. Function may deteriorate with eventual renal failure in the later
stages of the disease. Radiography may demonstrate a wavy renal outline with
tracks of contrast, ring shadows as a result of sloughing of papillae, and an egg-
in-a-cup appearance characteristic of renal papillary necrosis.

Scenario 3
A 75-year-old woman is referred with painless microscopic haematuria. She has also
noticed a bloody discharge staining her underwear. On examination, there are no
abdominal masses, inspection of the perineum reveals a red mass at the urethral
meatus. Intravenous urogram and cystoscopy are normal.

N - Urethral caruncle « CORRECT ANSWER


N – Urethral caruncle
A urethral caruncle is an inflammatory tumour, 1–1.5 cm in diameter, of the
urethral meatus in women, most frequently in the <st1:time ">6 o’clock position.
They are very vascular and covered with transitional epithelium. Although
frequently asymptomatic, they may give rise to spotting and microscopic
haematuria.
Oestrogen deficiency is implicated in the aetiology. The diagnosis is clinical,
although failure to respond to oestrogen therapy should prompt excisional
biopsy to exclude a more sinister pathology.
The Answer
 Comment on this Question
Haematuria may occur as a result of renal, ureteric, bladder, prostatic and
urethral causes. The following pathological processes are usually implicated:
trauma, infection, tumours, or stones. In addition, bleeding diatheses may
manifest as haematuria. When investigations fail to identify a cause, medical
disorders of the kidney should be sought by a nephrologist.

14

Theme: Staging of bladder tumours


A T3aG2
B TisG3
C TisG1
D T3bG2
E T4G3
F T4G1
G TaG2
For each of the cases below, choose the most appropriate staging from those
above. Each may be used once, more than once or not at all.
Scenario 1
Well-differentiated in situ carcinoma

C - TisG1 « CORRECT ANSWER


TisG1

Scenario 2
Moderately differentiated tumour invading the perivesical fat
D - T3bG2 « CORRECT ANSWER
T3bG2

Scenario 3
Poorly differentiated tumour invading the pelvis
E - T4G3 « CORRECT ANSWER
Staging of bladder cancer
Tumour staging
Tis Carcinoma in situ
Ta Papillary non-I-IV carcinoma
T1 Tumour invades sub-epithelial connective tissue (through lamina propria)
T2 Tumour invades muscle
T2a Tumour invades superficial muscle (inner half)
T2b Tumour invades deep muscle (outer half)
T3 Tumour invades peri-vesical tissue
T3a Microscopically
T3b Macroscopically (extra-vesical mass)
T4 Tumour invades adjacent structures
T4a Invades prostate, uterus, or vagina
T4b Invades pelvic wall or abdominal wall
Lymph node staging
Nx Cannot be assessed
N0 No nodes involved
N1 Single lymph node metastasis <2cm
N2 Single lymph node 2-5 cm or multiple nodes none >5cm
N3 Lymph node metastais >5cm
Metastases staging
Mx Cannot be assessed
M0 No distant metastases
M1 Distant metastases present
Histological grading of bladder tumours is:
G1 Well differentiated
G2 Moderately differentiated
G3 Poorly differentiated

15

A 52-year-old male presents to his GP with a swelling in his scrotum. This is


painless, but is becoming a nuisance due to its increasing size. On examination
there is a diffuse swelling of the scrotum, and it is not possible to distinctly
palpate the testicle. When using a pen-torch – it glows red.
What is the most likely cause?
Select one answer only
Epididymo-orchitis
Hydrocoele « CORRECT ANSWER
Hernia
Spermatocoele
Testicular tumour

 The Answer
 Comment on this Question
A hydrocoele consists of fluid accumulating within the tunica vaginalis and can
occur secondary to a patent processus vaginalis in neonates, (congenital), or be
idiopathic in origin, (primary). In a hydrocoele it is often difficult to palpate the
testicle separately. They transilluminate well to produce a red glow when a pen
torch is used.

16

Theme: Bladder outflow obstruction


A Benign prostatic hyperplasia (BPH)
B Bladder neck dyssynergia
C Bladder neck stone
D Bladder neck tumour
E Carcinoma of the prostate
F Clot retention
G Detrusor-external sphincter dyssynergia (DESD)
H Neuropathic bladder
I Pelvic mass/tumour
J Pelvic organ prolapse
K Previous bladder neck suspension surgery
L Urethral diverticulae
M Urethral dysfunction
N Urethral stone
O Urethral stricture
The pathologies above can all contribute to bladder outflow obstruction. For the
following scenarios please pick the most appropriate cause from the list. The
items may be used once, more than once, or not at all.

Scenario 1
A 52-year-old man presents to the urology outpatient department complaining of a
progressively ‘poor stream’ over the past few weeks. He tells you that he was recently
an in-patient, having undergone coronary artery bypass graft surgery, and that he
required an in-dwelling catheter for 6 weeks after suffering complications from the
surgery post-operatively. Before this he had had no urinary symptoms at all.

O - Urethral stricture « CORRECT ANSWER


O – Urethral stricture
This gentleman is likely to have developed a urethral stricture. Strictures can
result from an inflammatory process or trauma. Historically, gonorrhoea was a
common cause of stricture, but nowadays they tend to arise secondary to
traumatic urethral instrumentation (eg long-term catheterisation, cystoscopy, or
as a result of large-bore resectoscope use in transurethral resection of the
prostate). Prolonged urethral catheterisation, such as in this case, can lead to
stricture. This is particularly true in this scenario when, following coronary
bypass grafting, urethral ischaemia in a patient with cardiovascular disease may
be an exacerbating factor.

Scenario 2
A 73-year-old man finally presents to his general practitioner (GP) after an 18-month
history of urinary symptoms. He was previously too embarrassed to discuss his
worsening hesitancy, postmicturition dribbling and nocturia. You later see him in the
outpatients on his GP’s referral. On examination his abdomen is unremarkable and his
prostate appears smoothly enlarged. Prostate specific antigen 5.0 ng/ml.

A - Benign prostatic hyperplasia (BPH) « CORRECT ANSWER


A – Benign prostatic hypertrophy (BPH)
The presenting symptoms and prostate-specific antigen (PSA) result in this
clinical picture point to a diagnosis of BPH.
Lower urinary tract symptoms (LUTS) can vary between irritative storage
symptoms (frequency, urgency, nocturia) and voiding symptoms (hesitancy,
poor flow, intermittent flow and post-micturition dribbling). The PSA is within
normal limits for age (age-specific value for 70+ can be up to 6.5 ng/ml), although
in BPH, because of the increased prostatic volume, the level can be higher,
making it more difficult to rule out prostatic cancer. Management can initially be
via drug therapy (a-blockers or 5a-reductase inhibitors, eg Finasteride) but
ultimately, if patients remain symptomatic, they will be offered a transurethral
resection of the prostate (TURP).

Scenario 3
A 58-year-old man presents to The Emergency Department complaining of severe
suprapubic pain and an inability to pass urine for the preceding 12 h. Prior to that he
mentions he saw traces of blood in his urine. He tells you that he was recently
discharged from hospital following a transurethral resection of bladder tumour
procedure.

F - Clot retention « CORRECT ANSWER


F – Clot retention
The cause of this man’s acute retention is clot within the bladder. His history of
frank haematuria on a background of surgery for transitional cell cancer
resection (following transurethral resection of bladder tumour) makes this the
clear diagnosis. Immediate management will require the insertion of a rigid
three-way irrigating catheter (a normal flexible Foley catheter is likely to obstruct
with clot after a short period). This allows perfusion of the bladder with a normal
saline irrigation fluid until the bleeding has settled. More persistent bleeding may
require repeat rigid cystoscopy and diathermy to the affected area. Obviously a
full set of bloods should be drawn and transfusion should be undertaken as
necessary.

Scenario 4
A 35-year-old professional show jumper who had sustained a spinal cord injury after a
fall from his horse 1 month prior, is seen in the urology outpatients. On leaving hospital
after his initial injury he was reluctant to have a long-term urinary catheter and it was
agreed that he should try intermittent self-catheterisation (ISC) under the supervision of
the community continence nurse. He admits to you that he has been a little
unenthusiastic with the catheterisation in the past week (only emptying once or twice a
day) and had been experiencing some leaking in between attempts. On examination
he has a distended bladder. On siting a catheter, approximately 800 ml of clear urine
drains rapidly. A voiding cystourethrogram shows a voluminous bladder and every time
the patient tries to pass urine a narrowing is observed in the urethra between the
prostatic and bulbar urethra. Urea 16 mmol/litre; creatinine 240 µmol/litre.

G - Detrusor-external sphincter dyssynergia (DESD) « CORRECT ANSWER


G – Detrusor-external sphincter dyssynergia (DESD)
The higher centre for co-ordination of bladder with urethral function lies within
the pons and is known as the pontine micturition centre (PMC). The cell bodies
of the parasympathetic motor fibres to the detrusor muscle (S2–S4) and somatic
motor fibres innervating the striated urethral sphincter (S2–S4) are located in the
sacral spinal cord. They receive descending impulses from the PMC, which is
therefore responsible for ensuring that co-ordinated contraction of the bladder
and relaxation of the sphincter occur simultaneously, allowing normal voiding. In
DESD the PMC is disconnected from the sacral spinal cord (classically by spinal
cord injury) and patients lose the appropriate bladder–sphincter
synchronisation. In a reversal of normal function, when their bladder contracts it
does so forcibly against a closed urethral sphincter and hence these patients
develop retention with very high bladder pressures leading to renal back
pressure and renal failure (note raised urea and creatinine). As the patient tries
to pass urine the voiding cystourethrogram shows the external sphincter
(positioned between the prostatic and bulbar urethra) continuing to contract
when it should be relaxing.

17

A 36-year-old male is being investigated for subfertility and on further


questioning reports a dragging sensation in his scrotum which is exacerbated
on standing. On examination the right testicle feels normal, but on the left
distended blood vessels are seen that feel like a bag of worms. However, on
lying they disappear.
What is the most likely diagnosis?
Select one answer only
Hydrocoele
Indirect inguinal hernia
Orchitis
Testicular tumour
Varicocele « CORRECT ANSWER

 The Answer
 Comment on this Question
A varicocele is caused by an incompetence of the valves between the left
testicular vein and left renal vein causing dilated veins of the pampiniform
plexus. These often present as a dull ache in the scrotum and feel like a ‘bag of
worms’ in the scrotum. They become more obvious when the patient stands up.
They can be treated if they are symptomatic or in cases where patients have
reduced fertility, (although this doesn’t seem to affect pregnancy rates), by
surgery or embolisation.

18

Theme: Disorders of the prostate


A Acute prostatitis
B Benign prostatic hyperplasia
C Carcinoma of the prostate
D Chronic prostatitis
E Granulomatous prostatitis
F Prostatic abscess
G Prostatodynia
From the list above, select the most likely diagnosis for the following patients
with disorders of the prostate. The items may be used once, more than once, or
not at all.

Scenario 1
A 72-year-old diabetic gentleman is referred for assessment of lower urinary tract
symptoms. He reports worsening frequency of micturition and nocturia. He has also
noticed difficulties initiating micturition, and terminal dribbling. Abdominal examination
is normal, digital examination per rectum reveals a non-tender, enlarged prostate
gland. Urinalysis reveals microscopic haematuria. The following investigation results
are available: haemoglobin 13.4 g/dl, white cell count 5.4 x 10 9/litre, platelets 254 x
109/litre, Na+ 132 mmol/litre, K+ 5.3 mmol/litre, urea 9.8 mmol/litre, creatinine 165
µmol/litre, prostate-specific antigen 4.2 ng/ml.

B - Benign prostatic hyperplasia « CORRECT ANSWER


B – Benign prostatic hyperplasia
Benign prostatic hyperplasia (BPH) is the commonest disease to affect middle-
aged men. The aetiology remains unknown, although some form of androgen
imbalance appears important. The gland is enlarged by nodules of variable size
arising in the inner (peri-urethral) portion. Enlargement of the inner zone of the
gland tends to produce atrophy of the outer gland, which forms a pseudocapsule
to the prostate. Consequently, smooth enlargement of the gland is characteristic
of BPH on digital examination. Microscopically, these nodules are composed of
proliferating glands and fibromuscular stroma. The relative obstruction of
urinary flow produces the classical ‘obstructive’ symptoms associated with BPH,
namely, hesitancy, poor flow and terminal dribbling. Such symptoms are
relatively successfully addressed by surgical ‘decompression’ (eg transurethral
resection of the prostate). However, the bladder detrusor compensates for
increased outflow resistance with muscular hypertrophy and an increase in
collagen, resulting in trabeculation. Trabeculation is asymptomatic but the
detrusor becomes increasingly irritable, giving rise to ‘irritative’ symptoms of
urinary frequency, nocturia and urgency. Such symptoms are not usually
addressed by surgery, and are best treated medically (eg anti-cholinergics). With
progressive obstruction, chronic retention of urine secondary to incomplete
bladder emptying may precipitate infections, and in the longterm may lead to
chronic renal failure. Patients may experience acute retention of urine.
Management includes a-blockers, 5a-reductase inhibitors and, in resistant, cases
transurethral or transvesical resection of the prostate.

Scenario 2
A 34-year-old gentleman presents to The Emergency Department with acute retention
of urine. He reports worsening dysuria over the last few days, associated with rigors
and night sweats. He is currently taking triple immunosuppression therapy following
heart and lung transplantation for cystic fibrosis. Vital observations: temperature
38.9°C, pulse rate 125/min, blood pressure 90/46 mmHg, respiratory rate 16
breaths/min, SaO2 98%. Digital per rectal examination reveals a tender boggy prostate
that is fluctuant.

F - Prostatic abscess « CORRECT ANSWER


F – Prostatic abscess
Effective treatment of acute prostatitis with antibiotics has reduced the
prevalence of prostatic abscesses. However, they may be seen in patients with
immunocompromise. The clinical picture is of systemic sepsis as a result of
abscess formation, and occasionally, outflow obstruction/retention of urine
when the collection is large, as in the case described. The initial treatment is
with parenteral antibiotics. Transurethral incision may be required to drain the
abscess if medical treatment is ineffective.

Scenario 3
A 65-year-old gentleman presents with a history of abdominal distension and pain, and
worsening constipation. He has also suffered from weight loss. On direct questioning
he also reports marked perineal and lumbar pain. Urinalysis reveals microscopic
haematuria. Urea & Electrolytes reveal evidence of renal failure: Na + 132 mmol/litre,
K+ 5.9 mmol/litre, urea 20.6 mmol/litre, creatinine 256 µmol/litre.

C - Carcinoma of the prostate « CORRECT ANSWER


C – Carcinoma of the prostate
Carcinoma of the prostate is usually an adenocarcinoma of acinar form. Its
aetiology remains unknown. Most arise in the peripheral part of the gland, and
so it is amenable to detection on rectal examination. Presentation may be
asymptomatic (incidental finding), with a palpable nodule on per rectum
examination, with LUTI’s (usually obstructive in nature), although given the
peripheral location of tumours there is usually considerable involvement of the
gland by the time outflow obstruction develops. Patients may also present with
disseminated metastatic disease (bone and/or perineal pain), or locally advanced
disease, encircling the rectum and causing mechanical obstruction of the large
intestine, as in the clinical scenario presented.

19

A 72-year-old male with known chronic kidney disease is noted to have a tumour
on his left kidney in the pre-operative workup for an abdominal aortic aneurysm.
It is decided to perform an open AAA repair and left nephrectomy as a combined
procedure. 
Which of the following tests best allows assessment of the relative function of
each kidney?
Single best answer - select one answer only
CT
DMSA scan « CORRECT ANSWER
IVU
Plain radiography
US

 The Answer
 Comment on this Question
Dimercaptosuccinic acid (DMSA) is an example of a static radioisotope study in
which images are taken of a radioisotope taken up and retained in the renal
tubules. DMSA is taken up by tubular cells in proportion to their function. This
can allow assessment of the relative function of each kidney.
It is worth noting that MAG3 (not given as an option in this question) is
becoming more commonly used than DMSA.

20

A 22-year-old female with a history of renal colic is referred from the GP urgently
with severe left loin pain radiating to her groin. On further questioning she has a
history of rigors and on examination looks unwell with a temperature of 38.5 oC.
Her urine dipstick shows blood 3+ nitrites and leukocytes. A CT scan shows a
7mm stone in the left pelviureteric junction with proximal hydronephrosis.
What would be appropriate management for this?
Select one answer only
Catheterisation with an irrigation catheter
Flexible cystoscopy
Insertion of a nephrostomy « CORRECT ANSWER
Laparoscopy
Open surgery

 The Answer
 Comment on this Question
This case demonstrates an obstructed and infected urinary tract and as such
constitutes a urological emergency with a danger of pyonephrosis developing
unless the obstruction is urgently relieved. A nephrostomy tube can be placed
percutaneously to allow drainage of the kidney. In addition to this other
measures includes IV antibiotics and fluid resuscitation. 

Flexible cystoscopy allows visualisation of the bladder, but not the ureters.

21

Theme: Testicular swellings

A Maldescended testis
B Epididymo-orchitis
C Testicular torsion
D Spermatocele
E Hydrocele
F Varicocele
G Teratoma of the testis

For each of the following situations, select the most likely answer from the
above list. Each option may be used once, more than once or not at all.
Scenario 1
A 21-year-old man presents with non-tender swelling in the scrotum and a mass in the
neck.

G - Teratoma of the testis « CORRECT ANSWER


Testicular tumours can cause supraclavicular lymphadenopathy.

Scenario 2
A 30-year-old man with bilateral peri-mandibular swelling presents with painful testes.

B - Epididymo-orchitis « CORRECT ANSWER


Epididymo-orchitis may occur secondary to mumps, though is usually a complication of
a urinary tract infection (UTI), prostatitis or urethritis. It may also be associated with
hydrocele or urinary tract tuberculosis.

Scenario 3
An 83-year-old man with a mass in the left flank presents with swelling and discomfort
in the left testis, which does not resolve on lying down.

F - Varicocele « CORRECT ANSWER


Varicocele (varicosity of the pampiniform plexus) occurs more frequently on the left
side, due to testicular vein drainage. The patient is elderly with a renal mass, most
likely he has a renal carcinoma causing renal vein obstruction and secondary
varicocele. The description is indicative of a grade 3 varicocele which would be visible
as a scrotal swelling and palpable when the patient is lying down.

22

Theme: Abdominal pain

A Renal adenocarcinoma
B Ureteric colic
C Pelviureteric obstruction
D Aortic aneurysm
For each of the scenarios given choose the most likely diagnosis. Each option
may be used once, more than once, or not at all.
Scenario 1
A 55-year-old man presents with haematuria, loin pain and a loin mass. He has lost
weight recently.

A - Renal adenocarcinoma « CORRECT ANSWER


The presentation of the patient in this case is classically that of renal carcinoma;
however, this triad of symptoms and signs occurs in only 30% of cases.

Scenario 2
A 22-year-old patient gets loin pain mainly in the morning after drinking four cups of
coffee.
C - Pelviureteric obstruction « CORRECT ANSWER
Pelviureteric obstruction may be exacerbated by increased fluid intake.

Scenario 3
A 70-year-old patient presents with loin pain, with pulse 120/min and BP 80/60.
D - Aortic aneurysm « CORRECT ANSWER
Any male patient aged over 55 years who presents with loin pain should be
suspected of having a leaking abdominal aortic aneurysm (AAA) till proven
otherwise, because AAAs are more common in this age group than urinary
stones.

23

A 21-year-old male presents with pain in his left scrotum which has been
increasing for the past 2 days. He is systemically well and on examination his
abdomen is soft and non-tender. On palpation the left testicle is slightly tender
with no palpable lumps, but he is very tender slightly above and behind the
testicle.
What is the most likely diagnosis?
Select one answer only
Epididymo-orchitis « CORRECT ANSWER
Hydrocoele
Inguinal hernia
Testicular tumour
Torsion

 The Answer
 Comment on this Question
Epididymo-orchitis is an inflammatory condition of the testicle and epididymis
secondary to infection. This may occur as a result of a viral infection such as
mumps, or bacterial infection due to a UTI or sexually transmitted disease e.g.
chlamydia or gonorrhoea. The epididymis is often swollen and tender. 

In a hydrocoele the testicle is often impalpable and it is usually painless. An


inguinal hernia extending into the scrotum would cause a lump and it is not
possible to ‘get above it’. Testicular tumours are often painless, palpable lumps
and in a torsion the testicle is often exquisitely tender.

24

Theme: Urine output

A Acute tubular necrosis


B Acute urine retention
C Blocked catheter
D Dehydration
E Ureteric obstruction
F Chronic renal failure

Match the most appropriate option from the list above to each clinical situation
described below. Each option may be used once only, more than once or not at
all.
Scenario 1
Post-operative emergency AAA repair in the intensive care unit. The patient is
haemodynamically stable with good peripheralcirculation, but his urine output is
fluctuating between 1 and 5ml/h.

A - Acute tubular necrosis « CORRECT ANSWER

Scenario 2
A woman who underwent an open cholecystectomy 24 h ago drops her urine output to
15 ml in 4 h. She is hypotensive.

D - Dehydration « CORRECT ANSWER

Scenario 3
A 68-year-old man who underwent a bilateral inguinal hernia repair has had no urine
output in 36 h. He is not catheterised.

B - Acute urine retention « CORRECT ANSWER


Scenario 4
A catheterised patient who is in the high-dependency unit with acute pancreatitis had a
previously good urine output. He is well hydrated. He has now been anuric for the last
2 h.

C - Blocked catheter « CORRECT ANSWER

 The Answer
 Comment on this Question
Acute tubular necrosis is a common cause of acute renal failure in hospital, and in the
post-AAA repair patient, especially in an emergency scenario, this is usually due to
systemic hypotension (e.g. due to haemorrhage).In the surgical patient with low urinary
output always ensure there is not an obstruction. This may be a blocked catheter or, if
the patient is not catheterised, acute retention. Acute retention is common in elderly
men. Also check hydration.The minimum urine output acceptable is 0.5 ml/kg/h, but
ideally it should be 1–2 ml/kg/h.

25

Theme: Renal tract calculi


A Conservative management
B Extracorporeal shock wave lithotripsy (ESWL)
C Nephrectomy
D Percutaneous nephrolithotomy (PCNL)
E Percutaneous nephrostomy
F Ureteroscopy

For each of the patients below, select the most appropriate treatment from the
above list. Each option may be used once, more than once, or not at all.
Scenario 1

A 24-year-old woman presents with intermittent right loin pain. A mid-stream urine
specimen (MSU) confirms microscopic haematuria. A plain radiograph shows a 1.2-cm
calculus in the region of the right kidney. An intravenous urogram (IVU) confirms that it
lies within the renal pelvis but is not causing obstruction.
B - Extracorporeal shock wave lithotripsy (ESWL) « CORRECT ANSWER
B – Extracorporeal shock wave lithotripsy (ESWL)
Stones measuring < 2 cm in diameter, that lie within the kidney, are usually
treated with ESWL. Percutaneous nephrolithotomy (PCNL) is used for a stone
bulk > 2 cm (or > 1 cm in the lower pole calyx). ESWL can be used afterwards to
residual fragments (called Steinstrasse, which have the appearance of a stone
street in the ureter). Stones in the lower pole calyx have poor clearance rates
and thus PCNL is the preferred option.

Scenario 2
A 45-year-old woman presents with a history of recurrent urinary tract infections (UTIs)
and chronic left loin pain. An ultrasound shows a large echogenic mass in the left
pelvicalyceal system. A plain kidney and upper bladder (KUB) demonstrates a
staghorn calculus. A dimercaptosuccinic acid (DMSA) scan shows differential split
function left : right, 9 : 91.

C - Nephrectomy « CORRECT ANSWER


C – Nephrectomy
A staghorn calculus in a functional kidney is treated with PCNL followed by
ESWL to remove residual fragments. However, in a patient with a 15% split
function,
the most appropriate treatment would be nephrectomy if the split function is <
15%.

Scenario 3
A 31-year-old man presents with colicky left loin pain. He is tachycardic, flushed and
has a temperature of 38.5 °C. An IVU shows an obstructing 3mm calculus in the mid-
ureter.
E - Percutaneous nephrostomy « CORRECT ANSWER
E – Percutaneous nephrostomy
Obstructed infected kidneys need immediate drainage by
percutaneous nephrostomy (from above). Insertion of a ureteric stent (from
below) is useful for preventing a stone causing a PUJ obstruction, for the
prophylaxis of stones > 1 cm prior to more definitive treatment and to keep
luminal patency after accidental or planned ureteric opening.

26
Theme: Hemiscrotal pain
A Acute epididymo-orchitis
B Chronic post-vasectomy pain
C Fournier’s gangrene
D Hydrocoele epidydimo-orchitis
E Inguinoscrotal hernia
F Orchitis appendage
G Referred pain
H Testicular injury
I Testicular torsion
J Testicular tumour
K Tuberculous
L Torsion of testicular appendage
M Varicocoele
The following patients all present with hemiscrotal pain. From the list above,
select the most likely diagnosis. The items may be used once, more than once,
or not at all.
Scenario 1
A 9-year-old boy attends The Emergency Department with a sudden onset of severe
pain in the left hemiscrotum, which is associated with lower abdominal discomfort.
There is no history of trauma. He is in pain but afebrile. Examination of the external
genitalia reveals normal position of both testicles, and no erythema or increased
temperature. The left testicle is exquisitely tender over a small area on the upper pole,
and there is marked thickening of the cord.

L - Torsion of testicular appendage « CORRECT ANSWER


L – Torsion of testicular appendage
There are four testicular appendages, which represent embryological remnants.
These are the appendix testis (hydatid of Morgagni); the paraepididymis (organ
of Giraldes); vasa aberrantia; and the appendix epididymis (pedunculated
hydatid). The appendix testis undergoes torsion in 90% of cases. Torsion of an
appendage is more common in boys under the age of 11 years than torsion of
the testis. Similarly, acute epididymoorchitis is unusual in young boys, unless
there is an anatomical abnormality. The onset of pain is acute, and there is
usually oedema of the cord. There may be an associated hydrocoele, making
palpation of the twisted appendage difficult. Treatment involves exploration (also
allowing absolute exclusion of testicular torsion) and removal of the appendage.

Scenario 2
A 58-year-old diabetic gentleman develops worsening scrotal pain following drainage
of a peri-anal abscess. He is unwell and has a temperature of 38.5°C. Examination of
his external genitalia reveals a painful, erythematous swollen right scrotum. The
swelling also appears to be extending into the right inguinal region, and to a lesser
degree, the left scrotum.

C - Fournier’s gangrene « CORRECT ANSWER


C – Fournier’s gangrene
This is necrotising subcutaneous infection of the scrotum. The initial description
by Fournier was in young healthy men, without obvious cause. Nowadays, the
condition is most commonly seen in middle-aged or elderly gentlemen; there are
frequently contributory factors such as local perineal/peri-anal or lower urinary
tract surgery and immunocompromise (eg diabetes mellitus).
Streptococci and Clostridium perfringens are usually implicated, and infection
results in vascular thrombosis, subcutaneous tissue necrosis and eventually
gangrene. The infection follows the same path as extravasation of urine,
spreading into the perineum and lower abdominal wall. Treatment involves
aggressive broad-spectrum antibiotic therapy, and emergency wide surgical
excision of affected tissue.

Scenario 3
A 33-year-old man presents to The Emergency Department with a 7-day history of pain
affecting the right hemiscrotum. On direct questioning, he reports symptoms of dysuria
preceding the onset of the pain. There is no history of trauma. On examination, his
temperature is 37.9°C, and his right scrotum appears enlarged compared to the left.
Palpation of scrotal contents reveals a tender thickening posterior to the testis, which
itself is relatively painless.

A - Acute epididymo-orchitis « CORRECT ANSWER


A – Acute epididymo-orchitis
The aetiology of this infection varies with age. In young boys it is usually a
bacterial infection associated with a structural abnormality of the lower
genitourinary system. In young men it is most commonly sexually transmitted in
origin, with Chlamydia and Neisseria gonorrhoeae commonly implicated. In
older men it usually relates to prostatism, chronic retention of urine and/or
instrumentation of the lower urinary tract. There is usually gradual onset of pain
and swelling over several hours or days. There may be associated lower urinary
tract symptoms. A swollen, tender epididymis may be palpable, but it may also
fuse with the testis, forming a large inflammatory mass. A careful history and
examination is crucial in establishing the correct diagnosis, and differentiating
the problem from torsion. There may be associated bacteriuria. Treatment is with
broadspectrum antibiotics, with the addition of doxycycline (to cover Chlamydia)
if an acquired cause is expected.

27

Theme: Scrotal swellings


A Acute epididymo-orchitis
B Acute haematocoele
C Chronic haematocoele
D Epididymal cyst
E Inguinal hernia
F Orchitis
G Primary hydrocoele
H Secondary hydrocoele
I Testicular seminoma
J Testicular teratoma
K Testicular torsion
L Tuberculous
M Varicocele
The above are all potential causes of a swelling in the scrotum. For the ensuing
clinical scenarios please pick the most appropriate answer from the list. Each
item may be used once, more than once, or not at all.

Scenario 1
A 16-year-old presents to his general practitioner (GP) complaining of a vague,
dragging sensation and aching in the left scrotum. The GP examines him lying flat and
cannot identify anything unremarkable within either hemi-scrotum. On standing,
however, there is a soft area of bulging swelling that appears in the left upper scrotum.

M - Varicocele « CORRECT ANSWER


M – Varicocele
This is a condition of varicosities of the pampiniform plexus of veins (ie varicose
veins of the spermatic cord) and is present in 15–25% of all men. They usually
manifest first in adolescence and are more common on the left. This is explained
by the venous drainage of the left testicular vein into the left renal vein at right
angles (the right testicular vein drains obliquely into the inferior vena cava).
Absent or incompetent valves at this junction with the left renal vein lead to back
pressure, and the formation of the varicocele. On examination at this early stage
and with the patient standing the varicocele is said to feel like a ‘bag of worms’
(Grade 2); it cannot be felt supine at this stage as the veins are empty. Usually
varicoceles are managed conservatively (close-fitting underwear, reassurance,
analgesia for testicular ache); however, troublesome varicosities can be treated
by radiological embolisation, or by surgical ligation of the testicular veins in the
inguinal canal.

Scenario 2
A 13-year-old boy presents to Casualty in the early hours of the morning complaining
of unbearable pain in his left scrotum. The onset was sudden, woke him from sleep
and caused him to vomit.

K - Testicular torsion « CORRECT ANSWER


K – Testicular torsion
The classical age, history and clinical presentation in this scenario clearly points
to testicular torsion. The commonest age for torsion is between 10 and 15 years
and the problem is very uncommon over the age of 25 years. The majority of
torsions occur spontaneously, often in the early hours of the morning; however,
some follow minor trauma (eg blow to the scrotum during sport or while
mounting a bicycle). In young sexually active men it may be difficult to
distinguish between an acute epididymo-orchitis and testicular torsion. Surgical
exploration is compulsory.

Scenario 3
A 47-year-old man presents to the urology outpatient clinic with a 3–4-year history of a
slowly enlarging, non-tender swelling in his right scrotum. On examination you note a
multilocular 2-cm swelling located at the upper, posterior pole of the right testis. It is
fluctuant, transmits a fluid thrill and transilluminates. The cord is easily palpated above
the bulge.

D - Epididymal cyst « CORRECT ANSWER


D – Epididymal cyst
Epididymal cysts are fluid-filled swellings connected to the epididymis and are
thought to be derived from the collecting tubules of the epididymis. Most occur
in men over the age of 40 years who complain of a slowly enlarging, non-tender
bulge in the scrotum. Clinically they are as described in the scenario above. The
differential diagnosis is that of a hydrocoele, but the epididymal cyst can easily
be distinguished because of its position above and behind the superior pole of
the testis. The fluid of a hydrocoele surrounds the testis and usually makes the
testis impalpable.

Scenario 4
A 27-year-old man attends The Emergency Department complaining of bilateral
testicular pain and swelling. He gives a 5-day history of fever and malaise and tells you
that he had some bilateral jaw swelling and pain that appears to be settling now. On
examination he has a temperature of 38.9°C; his testes are swollen, tender to
palpation and feel somewhat soft.

F - Orchitis « CORRECT ANSWER


F – Orchitis
This man’s history of fever, malaise and parotitis indicates infection with the
mumps virus (a systemic paramyxovirus). Orchitis occurs in up to 20% of post-
pubertal men that contract the virus, commencing 4–6 days after the onset of the
parotid gland swelling and lasting 7–10 days. Diagnosis is confirmed by the
rising titre of anti-mumps antibody. Treatment is supportive (bed rest, analgesia
and scrotal support). Mumps orchitis can be complicated by testicular atrophy. If
the orchitis is bilateral fertility may be impaired.

28

Theme: Benign Prostatic Hyperplasia


A Doxazosin
B Finasteride
C Radical prostatectomy
D Retropubic (open) prostatectomy
E Trial without catheter
F Transurethral resection of the prostate (TURP)
G Urethral catheterisation
For each of the patients below, select the most appropriate treatment from the
above list. Each option may be used once, more than once, or not at all.

Scenario 1
A 71-year-old man presents with acute urinary retention. On catheterisation his
residual volume was 800 ml. His creatinine concentration on admission was 350
mmol/l. Following management of a postobstructive diuresis the creatinine
concentration returned to 90 mmol/l. Digital rectal examination suggests a large
benign-feeling prostate. A transrectal ultrasound shows a prostate volume of 180 ml
with no hypoechoic areas.

D - Retropubic (open) prostatectomy « CORRECT ANSWER

Scenario 2
A 56-year-old man presents with moderate lower urinary tract symptoms. He has
persistent macroscopic haematuria. A digital rectal examination shows a large benign-
feeling prostate. MSU, cytology, an IVU and flexible cystoscopy were negative for
transitional cell carcinoma. He wishes to have another child in the near future.

B - Finasteride « CORRECT ANSWER

Scenario 3
A 59-year-old man presents with vague abdominal pain. An ultrasound showed
bilateral hydronephrosis with a post-micturition residual volume of 1500 ml. His serum
creatinine was normal.

G - Urethral catheterisation « CORRECT ANSWER

 The Answer
 Comment on this Question
The morbidity in patients with very large prostates (> 100 g) is less if open retropubic
prostatectomy is performed rather than a transurethral resection of the prostate
(TURP), as this reduces operation time and avoids excessive fluid absorption during
prolonged surgery.
Finasteride is a useful treatment in men with
 large (> 40 g) prostates.
 It also reduces prostatic bleeding,
 and carries a lower risk of retrograde ejaculation (and therefore less risk of
reduced fertility) than tamsulosin.
One must always warn a patient undergoing TURP of the risk of retrograde ejaculation
following the operation. In the last scenario, the patient has acute-on-chronic urinary
retention (note that acute retention is typically painful) with a large bladder residual
volume (1500ml) and bilateral hydrophrosis that needs urgent relief with a urethral
catheter. The cause of this retention (i.e. most commonly an enlarged prostate) should
subsequently be investigated and managed.

29

Theme: Renal tract pathologies


A Adenocarcinoma of the kidney
B Adenoma of the renal cortex
C Angioma of the renal artery
D Angiomyolipoma
E Nephroblastoma
F Neuroblastoma
G Papillary transitional cell tumour of the renal pelvis
H Renal tuberculosis
I Squamous cell carcinoma of the renal pelvis
J Transitional cell carcinoma of the bladder
K Transitional cell tumour of the ureter
For each of the following statements, select the most likely cause for renal tract
disease from the above list. Each option may be used once, more than once, or
not at all.
Scenario 1
A 55-year-old smoker presents with a dragging discomfort in his left loin. He also gives
a history of haematuria with occasional clot colic. On examination, a mass is felt over
the left loin and he has a left-sided varicocele.
A - Adenocarcinoma of the kidney « CORRECT ANSWER
A – Adenocarcinoma of the kidney
Adenocarcinoma of the kidney (hypernephroma; Grawitz’s tumour) affects more
males than females (2 : 1) and is more prevalent in patients over 40 years of age.
Risk factors include smoking, genetic factors, a high intake of fat, oil and milk,
and exposure to toxins, such as lead, cadmium, asbestos and petroleum
products. Clinical features include: a dragging discomfort in the loin and a triad
of haematuria (with occasional clot colic), flank pain (in 35–40%) and palpable
abdominal mass (in 25–45%). In men, a rapidly developing varicocele (most often
on the left) is a characteristic sign. This is because the left testicular vein drains
into the left renal vein, whereas the right testicular vein drains directly into the
inferior vena cava.
The patient may also manifest symptoms of hypertension, erythrocytosis and
hypercalcaemia.

Scenario 2
A 20-month-old baby boy is brought to the paediatric surgical clinic by his mother who
gives a history of failure to thrive, fever and occasional blood in the nappy. On
examination, a soft mass that does not cross the midline is palpable on the right side of
the abdomen.

E - Nephroblastoma « CORRECT ANSWER


E – Nephroblastoma
Nephroblastoma (Wilms’ tumour) is a malignant mixed tumour seen in infancy.
The tumours are usually solitary, soft, lobulated and are tan or grey in colour.
The infant may present with pyrexia, haematuria (blood in the nappy), failure to
thrive, and a non-tender abdominal (flank) mass. This mass does not cross the
midline which distinguishes it from neuroblastoma (which usually crosses the
midline), and is more nodular and irregular. Investigations include: blood count,
biochemical profile, ultrasound scan (to confirm the mass and to also to view the
other kidney), intravenous urogram (to give anatomical detail and an indication
of renal function) and renogram. It is usually treated by total nephrectomy or
partial nephrectomy (in children with bilateral disease) followed by radiotherapy.

Scenario 3
A 35-year-old man of Asian origin presents with evening rise of temperature, weight
loss, increased urinary frequency and painful micturition. Urine investigation reveals a
sterile pyuria.

H - Renal tuberculosis « CORRECT ANSWER


H – Renal tuberculosis
Renal tuberculosis commonly occurs in the 20–40-years age group; it is more
common in males than females (2 : 1) and the right kidney is affected more than
the left. The symptoms are an increase in the urinary frequency (both during the
day and night), painful micturition, renal pain and haematuria. Constitutional
symptoms are weight loss and a slight evening rise in temperature.
Chemotherapy (pyrazinamide, isoniazid, rifampicin) forms the basis of
management of genitourinary tuberculosis. The antituberculous drugs have high
urinary excretion rates.

Scenario 4
A 67-year-old smoker presents with a 5-month history of painless haematuria,
increased frequency of micturition and loss of weight. He worked in the dye industry
before his retirement.
J - Transitional cell carcinoma of the bladder « CORRECT ANSWER
J – Transitional cell carcinoma of the bladder
Transitional cell carcinoma of the bladder usually occurs over the age of 50 and
is more common in men. The aetiology of this condition includes cigarette
smoking (more than 20 cigarettes/day has 2–6 times risk of developing bladder
cancer), working in the aniline dye and rubber industry (because of excretion
of  -naphthyl-amine in the urine), schistosomiasis infestation of the bladder
and long-term catheterisation in paraplegic patients. Patients may present with
painless haematuria, dysuria, frequency and urgency of micturition.
Investigations include urine microscopy and culture (to rule out any infection)
and cystoscopy. Endoscopic resection of the mass followed by a 4–6 week
course of radiotherapy to the bladder and the pelvic side walls is useful in
treating most tumours. Combination regimens of cisplatin, methotrexate and
vinblastine (and adriamycin in some cases) are useful in the treatment of
metastatic disease.

30
A 71-year-old male presents to his GP with a history of haematuria. He denies
any associated pain and is generally well in himself. Examination is
unremarkable and his only past medical history of note is COPD secondary to a
long smoking history.
What is the most likely cause for the haematuria?
Select one answer only
Bladder cancer « CORRECT ANSWER
BPH
Prostate cancer
Renal colic
UTI

 The Answer
 Comment on this Question
Painless haematuria in any patient above the age of 50 should be considered
bladder Ca until proved otherwise. The main risk factor for transitional cell
carcinoma of the bladder in the UK nowadays is smoking. 

BPH and prostate Ca tend to affect the flow of urine and cause lower urinary
tract symptoms (LUTS) rather than haematuria. 
s

31

Theme: Investigation of urinary tract disorders


A Antegrade ureteropyelography
B Computed tomography (CT) with contrast
C Computed tomography (KUB (CT KUB)
D Cystourethroscopy
E DMSA (dimercaptosuccinic acid) scan
F Intravenous ureterogram (IVU)
G Kidney, ureter and bladder (KUB) X-ray
H MAG 3 scan
I Magnetic resonance imaging (MRI)
J Renal angiography
K Retrograde ureteropyelography
L Transrectal prostatic biopsy
M Transrectal ultrasound (TRUS)
N Ultrasound kidneys, ureters and bladder
O Urethrography and cystography
P Urodynamic studies
The above are all examples of investigations used in the diagnosis of urinary
tract disorders. Please pick the most appropriate investigation for the following
clinical presentations/descriptions. The items may be used once, more than
once, or not at all.

Scenario 1
A 62-year-old man presents to The Emergency Department complaining of haematuria,
gnawing right loin pain and malaise. Abdominal examination does not reveal anything
grossly abnormal. Urine cytology and culture are normal. An ultrasound is performed
urgently and shows a complex right renal cyst with a calcified wall and a more solid
central component. The patient tells you that he has had a prior allergic reaction to an
injected contrast that he had in his 50s for some form of abdominal investigation. How
should he be further investigated?

I - Magnetic resonance imaging (MRI) « CORRECT ANSWER


I – Magnetic resonance imaging (MRI)
The standard primary investigation for haematuria and loin pain (with or without
a mass) is an ultrasound, which has already been undertaken for this patient. If a
renal cyst demonstrates a solid intracystic element or an irregular or calcified
wall, as in this case, it is regarded as potentially malignant and requires further
imaging for staging. This is usually completed by computed tomography (CT)
scanning of the chest and abdomen, with intravenous contrast administration. In
patients who have a prior history of reaction to contrast, MRI is the chosen
modality for staging assessment. In the staging of patients with renal carcinoma
MRI appears more accurate in delineating inferior vena cava or renal vein
involvement, compared with CT.

Scenario 2
A 50-year-old woman who is an inpatient on the Urology Ward recently had a left-sided
percutaneous nephrostomy tube sited for pyonephrosis. This was performed as an
emergency procedure on her admission with sepsis and left loin pain, following an
ultrasound KUB showing left-sided hydronephrosis. Urea 15 mmol/litre; creatinine 205
µmol/litre. What further imaging should be undertaken to assess the cause of her
infected kidney?

C - Computed tomography KUB (CT KUB) « CORRECT ANSWER


C – Computed tomography KUB (CT KUB)
Although this woman has a nephrostomy tube sited and this could be used for
antegrade contrast studies of her urinary tract, she has raised urea and
creatinine that could be exacerbated by contrast use. CT KUB has the advantage
in this scenario of allowing rapid visualisation of the renal tract without the need
for contrast injection. It has a high sensitivity for ureteric stones but is generally
second-line to intravenous urography in women of child-bearing age. If no renal
tract stone is identified, CT KUB has the advantage over intravenous urogram,
and retrograde or antegrade ureteropyelography, in that it can demonstrate other
intra-abdominal pathologies that might be causing upper tract obstruction.

Scenario 3
A 50-year-old smoker presents to the urology outpatients after referral by his general
practitioner for painless haematuria. His prostate feels smooth and of normal size and
he denies any lower urinary tract symptoms. All relevant blood tests and urine samples
have been taken and show nothing of significance other than confirming haematuria.
You need to arrange the next investigation.

D - Cystourethroscopy « CORRECT ANSWER


D - Cystourethroscopy
For patients of this age group (and beyond) with painless haematuria, bladder
cancer must always be excluded. Urgent cystourethroscopy (specifically a
‘flexible cystoscopy’) should be organised to visualise the bladder and prostatic
urethra (i.e. the common sites of bleeding lesions). This may organised together
with an USS of the renal tract

32

A 17-year-old female is referred to the Urology team after an ultrasound shows


hydronephrosis on the left on the background of recurrent urinary tract
infections.
What is the best technique to assess if there is obstruction in the urinary tract?
Select one answer only
CT
IVU
MAG 3 study « CORRECT ANSWER
Micturating Cystourethrogram
MRI

 The Answer
 Comment on this Question
Technetium-99m mercaptoacetyltriglcine (MAG3) is used in dynamic
radioisotope studies of the function of a kidney over a period of time.
Progressive uptake of the isotope may occur in a dilated, but not obstructed
system. Intravenous injection of frusemide can then help with rapid clearance to
demonstrate if any obstruction is present.
As MAG3 has a faster clearance and smaller volume of distribution, it is
replacing Di-ethylene-triamine-penta-acetic acid (DTPA) in diuretic renography.

Micturating cystourethrography is done to determine the presence of


vesicoureteric reflux (VUR). Following introduction of contrast into the bladder
via a catheter- dynamic X ray studies allow the assessment of any reflux up the
ureter. 

33

A 17-year-old female is referred to the Urology team after an ultrasound shows


hydronephrosis on the left on the background of recurrent urinary tract
infections.
What is the best technique to assess if there is obstruction in the urinary tract?
Select one answer only
CT
IVU
MAG 3 study « CORRECT ANSWER
Micturating Cystourethrogram
MRI

 The Answer
 Comment on this Question
Technetium-99m mercaptoacetyltriglcine (MAG3) is used in dynamic
radioisotope studies of the function of a kidney over a period of time.
Progressive uptake of the isotope may occur in a dilated, but not obstructed
system. Intravenous injection of frusemide can then help with rapid clearance to
demonstrate if any obstruction is present. As MAG3 has a faster clearance and
smaller volume of distribution, it is replacing Di-ethylene-triamine-penta-acetic
acid (DTPA) in diuretic renography.

Micturating cystourethrography is done to determine the presence of


vesicoureteric reflux (VUR). Following introduction of contrast into the bladder
via a catheter- dynamic X ray studies allow the assessment of any reflux up the
ureter. 

34

THEME: Scrotal swellings

A Epidydimal cyst
B Hydrocele
C Varicocele
D Scrotal haematoma
E Testicular tumour

For each of the following clinical scenarios select the most likely answer from
the list above. Each option may be used once, more than once or not at all.

Scenario 1
A young man presents to the clinic with a ‘dragging sensation’ in his scrotum. On
palpation the scrotum feels like a ‘bag of worms’.

C - Varicocele « CORRECT ANSWER


A varicocele is a varicosity of the pampiniform plexus. The left side is more commonly
affected than the right. There is a rare association with carcinoma of the left kidney.
Scenario 2
A patient has a vasectomy. He presents a day later with bruising and swelling of his
scrotum.

D - Scrotal haematoma « CORRECT ANSWER


Scrotal haematoma is a complication of operations on the testis, cord or scrotal
structures. It can also occur after trauma. Treatment is analgesia and scrotal support.

Scenario 3
An elderly man presents with a large painless scrotum. The scrotum transilluminates.

B - Hydrocele « CORRECT ANSWER


A hydrocele occurs when fluid collects in the tunica vaginalis. Testicular tumours can
present as hydroceles, so ultrasound of the scrotum is recommended. They can be
repaired surgically with Jaboulay’s procedure – the sac is incised longitudinally,
everted and approximated behind the cord.

Scenario 4
A young man presents with a painless hard lump in his testicle.

E - Testicular tumour « CORRECT ANSWER


Testicular tumours are the most common malignancy in young men. They are
associated with maldescent of the testicle. The most common types are seminomas,
followed by non-seminomatous germ cell tumours. Primary treatment is orchidectomy.

Scenario 5
On routine examination, a patient is noted to have a swelling above and behind his
testicle.

A - Epidydimal cyst « CORRECT ANSWER


Epididymal cysts are generally painless, but patients sometimes complain of
discomfort. They can be confirmed by ultrasound. No treatment is necessary.

35
Theme: Transitional cell carcinoma
A Cystoscopy
B Intravesical BCG
C Intravesical mitomycin
D Methotrexate, doxorubicin, cyclophosphamide (M-VAC) chemotherapy
E Nephrectomy
F Nephroureterectomy
G Radical cystectomy
H Transurethral resection of bladder tumour
For each of the patients below, select the most appropriate treatment from the
above list. Each option may be used once, more than once, or not at all.

Scenario 1
A 64-year-old man presents with haematuria. An IVU shows normal upper tracts with a
filling defect in the bladder. Flexible cystoscopy confirms a tumour.

H - Transurethral resection of bladder tumour « CORRECT ANSWER


H – Transurethral resection of bladder tumour

Scenario 2
A 58-year-old woman with a history of superficial bladder cancer is found to have an
irregular filling defect in the right renal pelvis. CT confirms a solid mass.

F - Nephroureterectomy « CORRECT ANSWER


F – Nephroureterectomy

Scenario 3
A fit 55-year-old man presents with haematuria. Investigations reveal a bladder tumour.
Transurethral resection of bladder tumour (TURBT) shows a muscle-invasive bladder
cancer (stage T2) and EUA confirms the bladder is mobile. CT scanning shows three
2–3-cm pelvic lymph nodes.
D - Methotrexate, doxorubicin, cyclophosphamide (M-VAC) chemotherapy «
CORRECT ANSWER
D – M-VAC chemotherapy
 The Answer
 Comment on this Question
The treatment (in most cases) of a bladder tumour is a transurethral resection (TURBT)
of the polypoid part of the tumour and a biopsy to stage muscle invasion. If it is found
to be stage T2–T4a, one should perform cystectomy ± radiotherapy, plus
chemotherapy if preferred.The standard treatment of a transitional-cell carcinoma in
either the kidney or ureter is a nephroureterectomy, as these tumours are often
multifocal and surveillance of a ureteric stump is difficult. A cystectomy is
contraindicated if enlarged pelvic lymph nodes are detected preoperatively.

36

Theme: Renal presentations


A Acute renal failure
B Acute tubular necrosis
C Focal segmental glomerulosclerosis
D Gram-negative sepsis
E Membranous glomerulonephritis
F Renal abscess
G Renal calculi
H Renal papillary necrosis
I Renal tubular acidosis
J Renal tubular injury
For each of the following statements, select the most likely cause for the renal
disease from the above list. Each option may be used once, more than once, or
not at all.

Scenario 1
A 32-year-old man presents to the Emergency Department with sudden excruciating
right-sided abdominal pain of 4 hours duration. The pain is radiating from the right side
of his ribs towards the groin. Urinalysis reveals red blood cells.

G - Renal calculi « CORRECT ANSWER


G – Renal calculi
Patients with renal calculi present with sudden severe flank or abdominal pain
which may radiate to the scrotum or labia and/or into the ipsilateral
costovertebral angle. The patient is restless and inconsolable (unlike in acute
appendicitis and perforated viscera where he or she lies still). Urinalysis usually
shows red blood cells. White blood cells may be seen if there is an associated
infection. The development of calculi may be the result of altered metabolism
and excretion of calcium, uric acid, cystine or oxalate. The calculi usually
consist of the above elements either on their own or in combination.
Investigations for suspected renal calculi include: plain radiography kidneys and
upper bladder (KUB), IVU, ultrasound or a CT urogram (CTU). Calcium oxalate
calculi make up about 90% of the stones. Since they are radio-opaque, they are
visible in plain radiographs. Uric acid stones are virtually radiolucent.

Scenario 2
A 61-year-old presents with vomiting and anorexia of 4 days duration. He has had a left
nephrectomy for chronic pyelonephritis 3 years ago and now suffers from recurrent
right renal calculi. His urea is 24 and creatinine is 461.

A - Acute renal failure « CORRECT ANSWER


A – Acute renal failure
The aetiology of acute renal failure could be classified into pre-renal (decreased
renal perfusion due to haemorrhage, dehydration, burns, sepsis, etc), renal
(nephrotoxic drugs, such as NSAIDs and angiotensinconverting enzyme
inhibitors (ACE inhibitors)) and post-renal (ureteric and lower urinary tract
obstruction) causes. This patient has developed acute renal failure secondary to
obstruction because of renal calculi (only one kidney is present). It is essential
to exclude obstruction as the cause for acute renal failure, particularly in
patients with a solitary kidney. Obstruction needs to be relieved either surgically
(nephrostomy / extracorporeal shock wave lithotripsy) or radiologically
(percutaneous) depending on the level and type of calculus and the patient’s
general health.

Scenario 3
A 46-year-old patient with AIDS is noted to have proteinuria, hypoalbuminaemia and
generalised oedema 1 week after a renal transplant. Renal biopsy reveals IgM deposits
in the glomerulus.
C - Focal segmental glomerulosclerosis « CORRECT ANSWER
C – Focal segmental glomerulosclerosis
Focal segmental glomerulosclerosis is a recognised complication of renal
transplantation. It has a higher incidence in intravenous drug abusers and in
patients with HIV infection or AIDS. The condition presents with proteinuria,
hypoalbuminaemia, oedema and hypercholesterolaemia. Biopsy of the kidney
reveals focal glomerular deposits of IgM. More than 50% of the patients progress
to chronic renal failure.

Scenario 4
A 67-year-old man undergoes nephrostomy to relieve hydronephrosis of his right
kidney. Four hours post-operatively, he develops rigors and pyrexia and his blood
pressure is 100/60 mmHg.

D - Gram-negative sepsis « CORRECT ANSWER


D – Gram-negative sepsis
This patient has developed Gram-negative sepsis as a result of instrumentation
of the renal tract. The common organisms include Escherichia coli and
bacteroides. Prophylaxis with an antibiotic such as gentamicin is usually
recommended before surgery or instrumentation of the renal tract. Immediate
management of Gram-negative sepsis entails antibiotics, oxygen and
intravenous fluids.

37

Theme: Scrotal swellings

A Hydrocele
B Encysted hydrocele of cord
C Varicocele
D Torsion of hydatid of Morgagni
E Epididymo-orchitis
F Inguinal hernia (inguino-scrotal hernia)
G Testicular tumour

For each of the patients described below, select the most likely diagnosis from
the list of options above. Each option may be used once, more than once or not
at all. You may believe that more than one diagnosis is possible but you should
choose the ONE most likely diagnosis.
Scenario 1
A 21-year-old man, who is very tall, presents to the surgical outpatient clinic with a left-
sided scrotal swelling. On examaination, it is not possible to get above the swelling.
The swelling is compressible and increases on standing.
C - Varicocele « CORRECT ANSWER
The classical history of a varicocele is the sensation of a bag of worms in the
scrotum. The varicosities are more prominent when the patient is standing and
disappear or decrease in size when the patient lies down. Approximately 98% of
varicoceles occur on the left; the reasons for this are that the left testicular vein
forms a more vertical angle with the left renal vein, and the left renal vein is
crossed and may be compressed by the pelvic colon.
YOUR ANSWER WAS INCORRECT
Scenario 2
A 18-year-old man presents to the emergency department with sudden onset of
testicular pain. On examination, the testis feels firm and irregular at the apex of the
scrotum.
G - Testicular tumour « CORRECT ANSWER
The history of onset of testicular tumour is varied but may be associated with
onset of sudden testicular pain. This diagnosis should be suspected if the testis
feels irregular. An urgent ultrasound is required.
YOUR ANSWER WAS INCORRECT
Scenario 3
A 54-year-old man presents to the surgical outpatient clinic with an inguino-scrotal
swelling. On examination, cough impulse is present.
F - Inguinal hernia (inguino-scrotal hernia) « CORRECT ANSWER
A scrotal swelling with a cough impulse is more likely to be an inguino-scrotal
hernia.
YOUR ANSWER WAS INCORRECT
Scenario 4
A 16-year-old boy presents to his general practitioner with a painless, long-standing
swelling in his right scrotum. The swelling is transilluminant.
A - Hydrocele « CORRECT ANSWER
A painless, long-standing swelling that transilluminates within the scrotum is
most likely to be a hydrocele.
38

A 76-year-old male is referred from his GP with symptoms of a poor stream and
hesitancy. On examination he has a smooth and enlarged prostate gland, and is
thought to have BPH. He is commenced on tamsulosin.
What type of drug is tamsulosin?
α blocker « CORRECT ANSWER
Androgen antagonist
β blocker
Gonadotrophin-releasing hormone agonist
Sα reductase inhibitor
YOUR ANSWER WAS INCORRECT

 The Answer
 Comment on this Question
Tamsulosin is an α1A –adrenoceptor antagonist and shows some selectivity for the
bladder and causes relaxation of the bladder neck and prostate capsule which may be
useful in BPH patients. Androgen anatagonists such as Cyproterone and Flutamide are
used against prostate tumours. Gonadotrophin-releasing hormone agonists such as
Goserelin can act to inhibit gonadotrophin release in certain circumstances and
therefore can be of use in Prostate Ca. Finasteride is a 5α-reductase inhibitor and
inhibits the conversion of testosterone into dihydrotestosterone in cells. It can be used
in BPH.

39

Theme: Urinary incontinence


A Acute retention of urine
B Chronic retention of urine
C Detrusor overactivity
D Detrusor hypotonia
E Faecal impaction
F Fistulation
G Functional
H Urethral diverticulum
I Urethral sphincter incompetence
J Urethral stricture
K Urinary tract infection
L Uterovaginal prolapse
The following patients all present with urinary incontinence. From the list above,
select the most likely diagnosis. The items may be used once, more than once,
or not at all.
Scenario 1
A 68-year-old woman complains of worsening urinary incontinence. She reports loss of
a trickle of urine when coughing or during physical exertion. She denies urinary
frequency and urgency. In the past she has had four normal vaginal deliveries. Urine
culture is normal. On examination, there is no evidence of urinary retention clinically,
vaginal examination excludes significant prolapse. There is objective loss of urine on
coughing.

I - Urethral sphincter incompetence « CORRECT ANSWER


I – Urethral sphincter incompetence
This has recently, and more accurately, been re-defined as ‘urodynamic stress
incontinence’. Consequently, it is a ‘urodynamic diagnosis’. It is defined as the
involuntary loss of urine when intravesical pressure exceeds the maximum
urethral closure pressure in the absence of detrusor overactivity. It occurs as a
result of incompetence of the urethral sphincter (hence its former name), which
can be the result of weakness in any component of the sphincter mechanism
(supporting structures, eg pubourethral and vesical ligaments), intrinsic
sphincter mechanism, or extrinsic sphincter mechanism (puborectalis). It is
more common in women, because of their anatomically shorter and straighter
urethra. Presentation is commonly with incontinence provoked by ‘stresses’ that
increase intraabdominal pressure (eg coughing, sneezing, abdominal masses
etc). In the first instance, treatment may be conservative
[physiotherapy/mechanical devices/pharmacological agents (oestrogens)] or
surgical (suspension/sling procedures/peri-urethral bulking agents).

Scenario 2
A 72-year-old man presents with symptoms of urinary incontinence. The loss is
associated with marked urinary urgency and is worse when the ‘weather is cold’. He
has a 15-year history of bladder outflow obstruction. He recently underwent
transurethral resection of the prostate, which has been associated with a deterioration
of his continence.

C - Detrusor overactivity « CORRECT ANSWER


C – Detrusor overactivity
Detrusor overactivity (DO) refers to objective contraction (spontaneous or on
provocation) during the filling phase of cystometry, while the patient is
attempting to inhibit micturition. It is, therefore, a urodynamic
(cystometrographic) diagnosis. The contractions precipitate urinary urgency,
and may result in leakage of urine (‘urge incontinence’). The pathophysiology of
DO remains poorly understood, and so it is termed idiopathic in the majority of
cases. However, it may occur secondary to neuropathic lesions (multiple
sclerosis/spinal injury/cerebrovascular accidents/Parkinsonism) or bladder
outflow obstruction in men, as in the scenario presented. In the case of chronic
obstruction, there is associated trabeculation (muscular hypertrophy and an
increase in collagen) and increasing bladder irritability. Management of DO
involves behavioural therapy (bladder drill), pharmacological manipulation (anti-
cholinergic agents/calcium-channel blockers/anti-diuretics/hormone
replacement therapy), and in severe refractory cases augmentation cystoplasty
or more rarely urinary diversion.

Scenario 3
A 48-year-old woman complains of 'constantly being wet - day and night' following a
radical hysterectomy and radiotherapy for cervical cancer. Urinalysis reveals the
presence of protein and leucocytes. Urodynamic investigations are normal.

F - Fistulation « CORRECT ANSWER


F – Fistulation
A history of constant, uncontrollable loss of urine in an otherwise mentally intact
individual should alert you to the presence of a fistula. Vesicovaginal, and less
commonly urethrovaginal, fistulae usually occur secondary to gynaecological
surgery in the developed world. By contrast, obstetric injury is the most
common aetiology in the developing world. Assessment of such patients is
clinical (examination under anaesthesia with dye insertion) and radiological
(micturating cystogram). Repair is surgical.

 The Answer
 Comment on this Question
For continence to exist, urethral pressure must exceed intravesical pressure at all
times. Urinary incontinence is defined by the International Continence Society as ‘the
involuntary loss of urine which is objectively demonstrable and a social or hygienic
problem’. It may be classified into urethral or extra-urethral conditions.Urethral causes
include

 urethral sphincter incompetence


 detrusor overactivity (uninhibited detrusor contractions)
 overflow incontinence (eg secondary to retention of urine/faecal
impaction/uterovaginal prolapse/drugs/urethral strictures/detrusor hypotonia etc)
 urinary tract infection
 urethral diverticulum
 functional (impaired cognition/intellectual function/immobility etc).

Extra-urethral causes include

 congenital abnormalities (eg ectopic ureter/bladder exstrophy)


 fistula formation (ureteric/vesical/vagina).

40

Theme: Renal calculi


A Percutaneous nephrolithotomy (PCNL)
B Extracorporeal shock wave lithotripsy (ESWL)
C Alkaline diuresis
D Nephrectomy
E Percutaneous nephrostomy
F Expectant therapy
For each of the following cases listed below, select the most likely single
treatment from the options listed above. Each option can be used once, more
than once or not at all.
Scenario 1
A 30-year-old pregnant woman (26 weeks) presents with septicaemia and abdominal
pain. Investigations reveal an obstructed right kidney due to a 2 cm calculus. She is
commenced on intravenous antimicrobials.
E - Percutaneous nephrostomy « CORRECT ANSWER
Percutaneous nephrostomy
The most appropriate treatment in this patient is to drain the kidney through a
percutaneous nephrostomy and administer antimicrobials.
YOUR ANSWER WAS INCORRECT
Scenario 2
A 40-year-old man presents with a left-sided renal colic. Intravenous urogram (IVU)
shows a 1 cm calculus in the upper third of his ureter. There is no complete
obstruction. His symptoms fail to resolve on conservative management.

B - Extracorporeal shock wave lithotripsy (ESWL) « CORRECT ANSWER


Extracorporeal shock wave lithotripsy (ESWL)
ESWL is now used to treat 90% of calculi that do not pass spontaneously.
YOUR ANSWER WAS INCORRECT
Scenario 3
A 20-year-old man presents with a renal colic secondary to a 1 cm cystine calculus.

C - Alkaline diuresis « CORRECT ANSWER


Alkaline diuresis
Cystine calculi dissolve in alkaline media. Other conservative measures include
penicillamine and methionine restriction.
YOUR ANSWER WAS INCORRECT
Scenario 4
A 30-year-old man presents to the casualty department with a right-sided renal colic.
An IVU shows a 4 mm calculus in the distal part of the ureter with no complete
obstruction.
F - Expectant therapy « CORRECT ANSWER
Expectant therapy
Calculi up to 5 mm in diameter usually pass out spontaneously. Non-steroidal
anti-inflammatory drugs (NSAIDs) provide effective analgesia.

Scenario 5
A 40-year-old woman is found to have a staghorn calculus in a non-functioning kidney.
D - Nephrectomy « CORRECT ANSWER
Nephrectomy
YOUR ANSWER WAS INCORRECT
Scenario 6
A 60-year-old man presents with frequent attacks of left-sided renal colic due to a 2.5
cm calculus in the renal pelvis. He has a cardiac pacemaker and is known to have a 6
cm aortic aneurysm.

A - Percutaneous nephrolithotomy (PCNL) « CORRECT ANSWER


Percutaneous nephrolithotomy (PCNL)
Cardiac pacemakers and abdominal aortic aneurysms are contraindications to
ESWL.

41

A 27-year-old male attends A&E following an episode of haemoptysis. He is a


non-smoker and was previously well. On examination his chest and abdomen
are unremarkable, and a small 1x1cm non-tender lump is found in his left testicle
which he has not previously noted. His chest x-ray shows multiple opacities in
the lung fields. He is thought to have a testicular tumour.
What is the most likely stage of this tumour?
Stage I
Stage II
Stage III
Stage IV « CORRECT ANSWER
Stage V
YOUR ANSWER WAS INCORRECT

 The Answer
 Comment on this Question
The Royal Marsden hospital staging system can be used for testicular tumours.
It stages them from I to IV:
Stag Details
e
I Tumour confined to testis
IM Rising concentrations of serum markers with no other
evidence of metastases
II Abdominal node metastases
IIA ≤2cm in diameter
IIB 2-5cm in diameter
IIC >5cm in diameter
III Supradiaphragmatic nodal metastases
ABC Node stage as defined in Stage II
M Mediastinal
N Supraclavicular, cervical or axillary
O No abdominal metastases
IV Extralymphatic metastases
Lung ≤3 metastases
L1 ≥3 metastases, all ≤2cm in diameter 
L2 ≥3 metastases, one or more of which are ≤2cm in
L3 diameter

42

Theme : Scrotal pain and swellings


A Epididymo-orchitis
B Hydrocele
C Inguinoscrotal hernia
D Mumps
E Testicular seminoma
F Testicular teratoma
G Testicular torsion
H Torsion hydatid of Morgagni
I Varicocele
For each of the following situations, select the most likely cause of scrotal pain
from the above list. Each option may be used once, more than once, or not at all.
Scenario 1
A 20-year-old man has a 24-hour history of severe left scrotal pain and swelling. There
is frequency of micturition and dysuria for the past few days. He has a temperature of
39 ºC. There are leukocytes in the urine and the WBC is 15 x 10 9/l.

A - Epididymo-orchitis « CORRECT ANSWER


A – Epididymo-orchitis
The history here is characteristic of an acute epididymo-orchitis. The preceding
urinary symptoms and raised WBC make the diagnosis most likely. The most
common causative organisms are the Gram-negative bacilli. Thus therapy is
most appropriately directed towards these.
YOUR ANSWER WAS INCORRECT
Scenario 2
A 30-year-old man gives a history of dull left-sided scrotal pain for several months. On
examination both testes are normal and you notice some veins on his scrotal skin.
There is, however, a left hemiscrotal swelling present on lying down and it is not
possible to get above this.

C - Inguinoscrotal hernia « CORRECT ANSWER


C – Inguinoscrotal hernia
The presence of veins on the scrotal skin here is a red herring. Varicoceles do
not cause large swellings in the hemiscrotum per se, but also disappear in the
recumbent position. The inability to get above the swelling is a clinical sign of an
inguinoscrotal hernia.
YOUR ANSWER WAS INCORRECT
Scenario 3
A 30-year-old man presents to his GP with swelling of his right–left hemiscrotum. He
has noticed a dull ache but feels otherwise well. The testis is slightly enlarged and feels
irregular in shape. Blood tests show raised β -HCG but normal α -fetoprotein.

E - Testicular seminoma « CORRECT ANSWER


E – Testicular seminoma
Testicular cancer usually presents in the 20–40-year age group. It may be
asymptomatic, but there is often a history of incidental trauma – the reason for
this is unknown. Seminomas most commonly occur in 30–40-yearolds. In
contrast, teratoma occurs in a younger age group (20–30 years). The  -
fetoprotein produced by yolk-sac elements is raised in teratomas but not in
seminomas. Trophoblastic cells secrete  -HCG and this may be present in
either tumour.
YOUR ANSWER WAS INCORRECT
Scenario 4
A 7-year-old boy presents with a swelling of his right hemiscrotum. He is in
considerable discomfort with a temperature of 37.5 ºC. On examination his pain is very
well localised to the upper pole of the testicle. A bluish hue can be seen through the
scrotal skin.

H - Torsion hydatid of Morgagni « CORRECT ANSWER


H – Torsion hydatid of Morgagni
The differential diagnosis here is that of testicular torsion and torsion of a
hydatid of Morgagni. The very localised tenderness and the presence of
the bluish discoloration would make the latter the more likely diagnosis.

43

A 22-year-old male is brought into A&E with severe left flank pain after being
kicked during a football game. He reports episodes of haematuria and a CT scan
is done. This shows the left kidney has a laceration through the corticomedullary
junction into the collecting system.
What grade is this renal injury?
Select one answer only
Grade 1
Grade 2
Grade 3
Grade 4 « CORRECT ANSWER
Grade 5

 The Answer
 Comment on this Question
The American Association for the Surgery of Trauma (AAST) has developed a
renal injury severity grading scale. This is outlined below:
Grade Description of injury
1 Contusion or non-expanding subcapsular haematoma
No laceration
2 Non-expanding perirenal haematoma
Cortical laceration <1cm deep without extravasation
3 Cortical laceration >1cm without urinary extravasation
4 Laceration through corticomedullary junction into collecting
system
or
Vascular: segmental renal artery or vein with contained
haematoma
5 Shattered kidney
or
Vascular: renal pedicle injury or avulsion

44

Theme: Scrotal swellings


A Acute epididymo-orchitis
B Chylocele
C Haematocele
D Lymph varix
E Papillary cystoadenoma
F Primary hydrocele
G Strangulated inguinal hernia
H Teratoma of testis
I Torsion of testis
J Varicocele
For each of the following statements, select the most likely cause of scrotal pain
from the above list. Each option may be used once, more than once, or not at all.
Scenario 1
A 25-year-old presents with a painless, scrotal swelling of 8 months’ duration which he
attributes to an injury while playing football. On examination, the swelling over the
testis is uniform, firm and smooth. There is loss of testicular sensation.
H - Teratoma of testis « CORRECT ANSWER
H – Teratoma of testis
Ninety per cent of testicular neoplasms are germ cell in origin and the majority
are malignant. Teratomas (seen in the 20–30-years age group) and seminomas
(seen in the 30–40-years age group) are the commonest tumours. Others
include: Sertoli cell tumours (causing feminising symptoms such as
gynaecomastia) and Leydig cell tumours (causing sexual precocity).
Approximately 10% of malignancies are found in undescended testes, even after
orchidopexy. Most men present with a painless swelling, although 10% may
present with an acutely painful testis. The usual presentation is a sensation of
heaviness in the testis and the groin. A history of trauma is usually given in
history. Testicular sensation is lost early. A small hydrocele may be present and
the spermatic cord may be thickened from malignant infiltration. Trans-scrotal
biopsies should never be performed and the initial management is
orchidectomy.
YOUR ANSWER WAS INCORRECT
Scenario 2
A 7-year-old presents with a sudden onset of severe pain in his right groin and lower
abdomen associated with vomiting. On examination, a thickened and tender spermatic
cord is palpable above the testis.

I - Torsion of testis « CORRECT ANSWER


I – Torsion of testis
Testicular torsion is common in children. Symptoms include sudden agonising
pain in the groin and lower abdomen associated with vomiting. Torsion of the
fully descended testis may be difficult to diagnose. A high-lying testis with
thickening of the tender twisted cord is observed. Elevation of the testis worsens
the pain (in contrast to epididymo-orchitis where elevation relieves the pain).
However, these clinical signs are relatively non-specific and surgical exploration
is mandatory if the diagnosis is suspected.
YOUR ANSWER WAS INCORRECT
Scenario 3
A 42-year-old tall, thin man of Asian origin presents with an aching pain and a left-
sided scrotal swelling of 18 months duration. On examination, the scrotum on the left
side hangs lower, cough impulse is present and the left testis is smaller than the right.
The swelling disappears on lying down.
J - Varicocele « CORRECT ANSWER
J – Varicocele
A varicocele is the varicose dilatation of the veins draining the testis. It is
common in tropical countries and usually seen in tall, thin men with a pendulous
scrotum. Varicocele is generally asymptomatic but at times may cause a vague
and dragging discomfort. The scrotum on the affected side hangs lower than the
other side. The cough impulse may be present (hence a differential for an
inguino-scrotal hernia). When the patient stands the varicocele feels like a ‘bag
of worms’. On lying down, the swelling disappears as the veins empty. In long-
standing cases, the affected testis is smaller because of atrophy. Varicoceles are
associated with infertility. The sudden appearance of a varicocele in a middle-
aged man should always raise the suspicion of retroperitoneal disease.
YOUR ANSWER WAS INCORRECT
Scenario 4
A 37-year-old presents with a right-sided scrotal swelling. On examination, the testis is
not palpable and there is dullness on percussion. It is possible to get above the
swelling.

F - Primary hydrocele « CORRECT ANSWER


F – Primary hydrocele
Primary hydrocele is mostly seen in the middle aged. The common presenting
complaint is a scrotal swelling. On examination, a cough impulse is absent, the
swelling is dull to percussion and it is clinically possible to ‘get above’ it (thus
distinguishing it from an inguino-scrotal hernia). The fluid of the hydrocele
surrounds the body of the testis, making the testis impalpable. A hydrocele is
fluctuant and trans-illuminates.

45

A 3-year-old boy is referred from his GP as his parents have noted some
haematuria and weight loss. On examination he is noted to have a visible left
sided abdominal mass.
What is the most likely cause for this?
Select one answer only
Meckel’s diverticulum
Nephroblastoma « CORRECT ANSWER
Neuroblastoma
Renal cell carcinoma
Transitional cell carcinoma of the left ureter
YOUR ANSWER WAS INCORRECT

 The Answer
 Comment on this Question
Nephroblastomas (or Wilm’s tumours) occur in children <5 years and are an
undifferentiated embryonic tumour. They are the commonest intra-abdominal
tumours in children under 10 years of age and are bilateral in 10%. Presentation
can be with failure to thrive, and a visible abdominal mass. Around one third will
have haematuria.

Neuroblastomas are tumours derived from neuroendocrine cells and arise from
neural crest elements of the sympathetic nervous system. They most commonly
arise from the adrenal glands, but can develop elsewhere. In most cases there
will be alterations in catecholamine levels. 

46

You are called to review a 74-year-old male on the orthopaedic ward who is
complaining of severe pain in the tip of his penis, following catheterisation for
urinary retention, following a total hip replacement. On examination the prepuce
is retracted with the glands swollen and oedematous.
What is the most likely diagnosis?
Select one answer only
Balanitis
Balanitis xerotica obliterans (BXO)
Paraphimosis « CORRECT ANSWER
Phimosis
Traumatic catheterisation
YOUR ANSWER WAS INCORRECT
 The Answer
 Comment on this Question
A paraphimosis occurs when the prepuce is retracted beyond the glans and
cannot be replaced. This results in the prepuce acting as a tight band and
preventing venous return which can result in swelling and oedema of the glans.
It can be caused by careless catheterisation in which the foreskin is not returned
to its original position. 

A phimosis occurs when the foreskin is tight making retraction difficult. Balanitis
is an acute inflammation of the foreskin and glans often caused by a bacterial
infection. Balanitis Xerotica Obliterans (BXO) is a condition in which white
plaques are present on the glans and prepuce making retraction of the foreskin
difficult. It is an indication for circumcision.

47

Theme: Renal masses


A Hydronephrosis
B Nephroblastoma (Wilm’s tumour)
C Peri-nephric abscess
D Polycystic disease
E Renal cell carcinoma
F Renal haematoma
G Solitary cysts
H Supernumerary kidney

From the list above, select the most likely diagnosis for the following patients
who all present with enlargement of the kidney on physical examination. The
items may be used once, more than once, or not at all.
Scenario 1
A 59-year-old man is found to have enlargement of his right kidney on physical
examination. Past history includes emergency repair of a ruptured abdominal aortic
aneurysm 5 years ago. Urinalysis is normal, and no malignant cells are seen on
microscopy. His Urea & Electrolytes are normal.
A - Hydronephrosis « CORRECT ANSWER
A – Hydronephrosis
Hydronephrosis refers to dilatation of the renal pelvis and calyces associated
with progressive atrophy of the kidney as a result of the obstruction of outflow
of urine. Such obstruction may affect the upper urinary tract (ie ureteric
obstruction), potentially resulting in unilateral hydronephrosis, or the lower
urinary tract (bladder outflow obstruction etc), which usually results in bilateral
hydronephrosis. Ureteric obstruction may be the result of congenital
pelviureteric junction obstruction, or intraluminal (stones/tumours) or intramural
(primary megaureter) pathology. It may occur secondary to extrinsic
compression in the retroperitoneum (eg malignant disease/inflammatory
disease/aneurysms/retroperitoneal fibrosis etc). Fibrosis around an aortic graft
may result in ureteric obstruction, as in the case described. There may be
associated impairment of renal function, particularly if the obstruction is
bilateral. Ultrasound scanning will confirm that hydronephrosis is the cause of
the renal enlargement. Treatment is directed towards the cause of the
obstruction.

Scenario 2
A 4-year-old boy is referred by his general practitioner with an enlarged left kidney. He
is otherwise asymptomatic and completely well. He is afebrile. Urinalysis reveals
microscopic haematuria. Urine cultures are negative.

B - Nephroblastoma (Wilm’s tumour) « CORRECT ANSWER


B – Nephroblastoma (Wilm’s tumour)
This tumour is the commonest intra-abdominal malignancy in children under the
age of 10 years. It is composed of primitive renal tubules, primitive renal
blastoma-like mesenchyme, and fibroblast-like spindled cells. The peak
incidence is between 2 and 4 years but cases may present during adolescence
or even adulthood. The child is usually well, unlike in cases of neuroblastoma
from which nephroblastoma must be distinguished. The commonest
presentation is with an abdominal mass. Haematuria, pain, hypertension and
intestinal obstruction may occur. Metastases occur to the liver. Treatment
involves surgical excision and aggressive chemotherapy and radiotherapy. Five-
year survival is in the order of 90%.

Scenario 3
A 39-year-old woman is referred for assessment by an The Emergency Department
Senior House Officer who has identified a tender right renal mass. She tells you that
she has been unwell for several days, when she has been anorexic, suffering with
night sweats and generalised malaise. Previously, she has noted frequency of
micturition. There is no history of trauma. She presented to The Emergency
Department with ureteric colic 5 days ago but was discharged because the pain
appeared to settle. Urinalysis reveals 3+ of blood and protein.

C - Peri-nephric abscess « CORRECT ANSWER


C – Peri-nephric abscess
Acute bacterial infections of the kidney form a spectrum ranging from relatively
simple infection resulting in acute cystitis/pyelonephritis through to peri-nephric
abscess formation. It is often difficult to clearly differentiate these on clinical
grounds alone because they all present with localised symptoms of infection
affecting the urinary tract, coupled with a degree of systemic effects (malaise,
rigors, pyrexia, raised white count etc). Severe infection of the urinary tract may
lead to pyonephrosis, particularly if the outflow of the kidney is obstructed by
the presence of stones, as alluded to in the case scenario presented. Such a
collection may discharge into the surrounding peri-renal tissue forming a peri-
nephric abscess. Radiology is essential in differentiating between the various
forms of acute infection. Treatment involves appropriate broad-spectrum
antibiotics, and percutaneous or open drainage as required. An obstructed
infected kidney is an indication for emergency nephrostomy.

48

Theme: Treatment options for renal/ureteric calculi

A Percutaneous nephrolithotomy
B Conservative management
C Percutaneous nephrostomy
D Extracorporeal shock-wave lithotripsy
E Nephrectomy

For each of the following case histories, select the most likely answer from the above
list. Each option may be used once, more than once, or not at all.
Scenario 1
A 32-year-old man has a 24-hour history of right ureteric colic. He is apyrexial and pain
controlled with simple analgesia, and imaging reveals a 3-mm distal right ureteric
calculus with no hydronephrosis.

B - Conservative management « CORRECT ANSWER


More than 95% of ureteric calculi of 5 mm or less will pass through without intervention.
Indications for intervention in patients with ureteric calculi include: severe pain; sepsis
(an obstructed infected system); or a large calculus that is very unlikely to pass.
Intervention is also indicated for patients with a single kidney.
Scenario 2
A 72-year-old woman has a history of recurrent urinary tract infections (UTIs). Imaging
demonstrates a large staghorn calculus in her right kidney with a normal-looking left
kidney. A renogram demonstrates 35% function on the right side.

A - Percutaneous nephrolithotomy « CORRECT ANSWER


Large staghorn calculi are best treated with a percutaneous nephrolithotomy. This
offers the best chance of stone clearance in patients with large stone burdens. The
recurrent UTIs are most likely to be due to the large staghorn calculus.

Scenario 3
A 40-year-old woman has a 3-day history of left ureteric colic. She is pyrexial and
tachycardic with an increased white cell count. Imaging reveals a 9-mm mid-ureteric
calculus on the left with severe left-sided hydronephrosis.

C - Percutaneous nephrostomy « CORRECT ANSWER


This patient clinically and radiologically has an obstructed infected system that requires
intervention. In the acute phase, the objective is to drain the kidney – the insertion of a
ureteric stent would be an appropriate course of action.

Scenario 4
A 58-year-old man has a history of recurrent UTIs. Imaging reveals a large left-sided
staghorn calculus with renographic evidence of a non-functioning left kidney and a
normal right kidney.

E - Nephrectomy « CORRECT ANSWER


In a patient with a large stone burden and a non-functioning kidney, the most
appropriate treatment option is a nephrectomy. A non-functioning kidney is one
demonstrated renographically to have a function of < 15%.

49

Theme: Imaging
A Cystogram
B DMSA scan
C DTPA scan
D IVU H Ultrasound
E Plain KUB
F Retrograde ureterogram
G CT KUB scan
For each of the patients below, select the most appropriate treatment from the
above list. Each option may be used once, more than once, or not at all.
Scenario 1
A 34-year-old obese man presents with a sudden onset of colicky right loin pain
radiating to his groin. He has microscopic haematuria. He has a history of severe
anaphylaxis with intravenous contrast. An ultrasound scan is unhelpful because of
obesity.

G - CT KUB scan « CORRECT ANSWER


G – CT KUB scan
A CT KUB scan (which does not involve the use of contrast, unlike a CT
urogram) is the gold standard for detecting renal tract calculi. This imaging
modality may be used when an IVU or ultrasound are contraindicated or
impractical.

Scenario 2
A 45-year-old woman has a right staghorn calculus on plain KUB. An IVU shows this
kidney fails to excrete contrast. An ultrasound scan shows the kidney has a thin
parenchyma without evidence of hydronephrosis.

B - DMSA scan « CORRECT ANSWER


B – DMSA scan
Staghorn calculi require DMSA imaging to ascertain the split function of the
kidneys, as DMSA is secreted by the kidney.

Scenario 3
A 22-year-old woman presents with a history of left loin pain shortly after drinking
alcohol. An ultrasound scan shows hydronephrosis with a normal calibre ureter. An IVU
shows a narrowing at the pelviureteric junction.

C - DTPA scan « CORRECT ANSWER


C – DTPA scan
DTPA scans are used to show which kidney is obstructed, as it is filtered by the
glomerulus. MAG-3 is filtered and secreted, and is now the most commonly used
isotope in imaging departments. PUJ obstruction has a trimodal distribution:
antenatal, teenagers (when they start drinking alcohol) and the elderly. A
diuresis in a patient with a PUJ obstruction worsens the pain and should arouse
suspicion of this condition.

50

Theme: Initial treatment for patients with lower urinary tract symptoms

A Transurethral resection of the prostate


B 5-reductase inhibitor
C Anticholinergic
D Urethral catheter
E Alpha-blocker

For each of the following situations, select the most likely answer from the
above list. Each option may be used once, more than once, or not at all.
Scenario 1
A 70-year-old man presents with nocturia, a good urine flow, frequency, urgency, and
on direct questioning an episode of urge incontinence.

C - Anticholinergic « CORRECT ANSWER


This man presents with a history of filling (irritative) symptoms suggestive of an
overactive bladder. Present guidelines from the International Continence Society
suggest initial treatment with an anticholinergic agent, if this is unsuccessful then
urodynamic studies should be performed.

Scenario 2
A 60-year-old, commercial airline pilot presents with a poor urine flow and hesitancy.
This man has an obstructed-looking flow rate trace and is keen to avoid surgery.

B - 5-reductase inhibitor « CORRECT ANSWER


This man presents with obstructive symptoms and is keen to avoid surgery. Typically, a
patient with symptoms such as these would be offered an alpha-blocker. However,
such a drug would be contraindicated for a pilot because of the recognised side-effect
of postural hypotension. An alternative is a 5-reductase inhibitor, although patients
should be informed that it may take several months before they notice any effect of the
drug.

Scenario 3
A 94-year-old man presents with a poor urine flow, hesitancy and a feeling of
incomplete bladder emptying, with bilateral moderate hydronephrosis on ultrasound
and a residual volume of 2 litres. His creatinine is elevated at 200 μmol/l.

D - Urethral catheter « CORRECT ANSWER


This patient presents in chronic retention of urine with bilateral hydronephrosis and
renal impairment. In the first instance this patient requires catheterisation, following
which his options include either a long-term catheter or a TURP.

Scenario 4
A 74-year-old man presents with a poor urine flow, hesitancy and an ultrasound
demonstrating a small bladder calculus.

A - Transurethral resection of the prostate « CORRECT ANSWER


The patient has symptoms suggestive of bladder outflow obstruction. The presence of
a bladder calculus, formed most commonly as a result of outflow obstruction due to an
enlarged prostate gland, is an indication for a TURP.

You might also like